New classes are starting soon - secure your spot today!

Mathcamp 2021 Qualifying Quiz

Go back to the Math Jam Archive

Join the Canada/USA Mathcamp staff for this exciting Math Jam, where they discuss solutions to the 2021 Mathcamp Qualifying Quiz! Mathcamp is an intensive 5-week-long summer program for mathematically talented high school students. The Qualifying Quiz is the centerpiece of the Mathcamp application each year, and the problems are designed to be fun, challenging, and thought-provoking. We invite anybody who's interested to come to this event, whether you applied to Mathcamp this year or just want to talk about some cool problems with Mathcamp instructors. It'll be even more fun for you if you bring your own solutions so you can participate in the discussion during the Jam!

Copyright © 2024 AoPS Incorporated. This page is copyrighted material. You can view and print this page for your own use, but you cannot share the contents of this file with others.

Facilitator: MiraBernstein

AngelBell 2021-05-19 18:55:32
Welcome to the Canada/USA Mathcamp Qualifying Quiz Math Jam! The Math Jam will begin at 7:00 pm ET (4:00 pm PT).
AngelBell 2021-05-19 18:55:38
If you are enrolled in a course and trying to attend it, then you should leave the classroom now, click Classroom in the Online School dropdown, and then choose the appropriate link on the next page.
AngelBell 2021-05-19 18:55:45
The classroom is moderated: students can type into the classroom, but only the moderators can choose a comment to drop into the classroom. So, when you send a message, it will not appear immediately, and may not appear at all.
vapodaca 2021-05-19 19:00:09
Hello and welcome to the Canada/USA Mathcamp Qualifying Quiz Math Jam!
vapodaca 2021-05-19 19:00:24
Before I introduce our guests, let me briefly explain how our online classroom works.
vapodaca 2021-05-19 19:00:35
This room is moderated, which means that all your questions and comments come to the moderators. We may share your comments with the whole room if we so choose.
vapodaca 2021-05-19 19:00:46
Canada/USA Mathcamp is an intensive five-week-long summer program for high-school students interested in mathematics, designed to expose students to the beauty of advanced mathematical ideas and to new ways of thinking. You can learn more about Canada/USA Mathcamp here: www.mathcamp.org
vapodaca 2021-05-19 19:00:52
Many AoPS instructors, assistants, and students are alumni of this outstanding program!
vapodaca 2021-05-19 19:01:43
Tonight we'll have Marisa Debowsky leading our discussion!
vapodaca 2021-05-19 19:01:47
Marisa Debowsky (MarisaD) is the Executive Director of Mathcamp. She's been teaching Topological Graph Theory and singing pop songs at Mathcamp every summer since 2006.
vapodaca 2021-05-19 19:02:02
And now, I'll hand it off to Marisa!
MarisaD 2021-05-19 19:02:15
Hi, everybody, and welcome to the annual Mathcamp Qualifying Quiz Jam!
MarisaD 2021-05-19 19:02:21
A big thanks as always to @AngelBell, @vapodaca, @rrusczyk, and the AoPS team for hosting us.
MarisaD 2021-05-19 19:02:33
My Mathcamp colleagues and I are here to talk about the Mathcamp 2021 QQ. To follow along, you should all have the quiz open in another window: https://www.mathcamp.org/qualifying_quiz/current_quiz/
MarisaD 2021-05-19 19:02:44
The Quiz problems are written by Mathcamp alumni, staff, and friends each year, and the solutions we'll be walking through today are a collaboration by lots of Mathcamp staff (with good ideas from the applicants, too!).
MarisaD 2021-05-19 19:02:49
If you applied this year, I strongly recommend having your own solutions open. That way, you can reply more quickly to the questions we ask of the room. And we're expecting you all to pitch in to the solutions!
MarisaD 2021-05-19 19:02:56
Students can use LaTeX in this classroom, just like on the message boards. Specifically, place your math LaTeX code inside dollar signs. For example, type:

We claim that \$\frac{1}{2} + \frac{1}{3} = \frac{5}{6}\$.

This should give you:

We claim that $\frac{1}{2} +\frac{1}{3} = \frac{5}{6}$.
MarisaD 2021-05-19 19:03:17
Also, as @AngelBell and @vapodaca pointed out: this chat room is moderated. That means your messages go only to us, and we will choose which to pass on, so please don't be shy to contribute and/or ask questions about the problems at any time (and we'll do our best to answer).
MarisaD 2021-05-19 19:03:28
Today, we'll just be talking about the Quiz. If you have questions about Mathcamp itself, or the application process, you'll find lots of info on our website (e.g., at http://mathcamp.org/gettoknowmathcamp/), or check out the AoPS Jam about the program and the application process from a few months ago: https://artofproblemsolving.com/school/mathjams-transcripts?id=572
MarisaD 2021-05-19 19:03:35
If we don't end up getting to your questions, feel free to post them on the Mathcamp forum on AoPS: http://www.artofproblemsolving.com/community/c135
MarisaD 2021-05-19 19:03:40
We've got a lot to cover in the next two hours, so let's get started! I'll turn it over to Assaf to kick us off with Problem #1.
safibn 2021-05-19 19:03:46
Hello there! I’m Assaf, and I’ll be presenting problem 1. This problem has to do with the circle of lies as described below:
safibn 2021-05-19 19:03:52
http://www.math.toronto.edu/safibn/circle_of_lies.png
safibn 2021-05-19 19:03:58
In other words, mentors always lie to campers, campers always lie to JCs, and JCs always lie to mentors. All other interactions are truthful.
safibn 2021-05-19 19:04:06
Fun fact: all of the people mentioned in this problem have actually been mentors, campers, and JCs. Some have even lied to me.
safibn 2021-05-19 19:04:22
Part (a): Sachi and Yuval have a conversation. Sachi says to Yuval, "At least one of us is a mentor." Yuval replies to Sachi, "At least one of us is a camper." What can you deduce about Yuval?
safibn 2021-05-19 19:04:38
There’s various approaches to solving this problem. We can use a truth table, or analyze all possible role pairs for Yuval and Sachi, or analyze whether Yuval or Sachi are lying. I’ll show the latter.
safibn 2021-05-19 19:04:52
First, notice that it’s impossible for both Yuval and Sachi to be lying.
safibn 2021-05-19 19:05:02
Sachi is lying: If Sachi is lying, then neither Yuval nor Sachi are mentors. What does this mean? *consults circle of lies*
squirrellover 2021-05-19 19:05:40
Yuval is a JC
Lilavigne 2021-05-19 19:05:40
sachi is a camper and yuvi is a jc
SHZhang 2021-05-19 19:05:44
Sachi is camper, Yuval is JC
safibn 2021-05-19 19:05:54
If Sachi is lying, then Sachi must be a camper and Yuval must be a JC
CircleInvert 2021-05-19 19:05:58
Sachi is a camper and Yuval is a JC
RoboBuilder 2021-05-19 19:05:58
Sachi is a camper and Yuval is a JC
safibn 2021-05-19 19:06:14
yuval is lying: if yuval is lying, then neither yuval nor sachi are campers. what does the circle of lies tell us?
rickiebosun 2021-05-19 19:06:53
Yuval is a JC and Sachi is a Mentor
w_aops 2021-05-19 19:06:53
Yuval is a JC and Sachi is a mentor
Peatato 2021-05-19 19:06:53
Yuval is JC and Sachi is a mentor
Lilavigne 2021-05-19 19:06:53
Yuval is a JC and Sachi is a mentor
m0bius 2021-05-19 19:06:53
Yuval is a JC and Sachi is a mentor
eshu_mittal 2021-05-19 19:06:53
Yuval is a JC, Sachi is a mentor
safibn 2021-05-19 19:07:06
If Yuval is lying then Sachi must be a mentor and Yuval must be a JC
safibn 2021-05-19 19:07:15
Neither Yuval nor Sachi are lying: In this case, one of them is a mentor, and one is a camper, but then one would lie to the other! This can’t happen
safibn 2021-05-19 19:07:22
In all possible cases, Yuval is a JC
safibn 2021-05-19 19:07:34
Part (b): You are a camper and you meet someone named Miranda at camp. What question can you ask Miranda about herself to determine whether she is a JC?
safibn 2021-05-19 19:07:45
Some answers were of the form: I, a camper, will say: “Your name is Miranda”, and if the universe will implode in a paradox if Miranda is a JC.
safibn 2021-05-19 19:07:55
We did not accept answers that imploded the universe in a paradox.
safibn 2021-05-19 19:08:07
What happens if you asked Miranda “Are you a JC?”
gordonhero 2021-05-19 19:08:39
JC would say yes if they are a JC, but yes if they are a mentor as well
safibn 2021-05-19 19:08:44
So that’s no good - it detects campers instead of JCs. What question would detect JCs?
safibn 2021-05-19 19:08:56
If we ask: “Are you a camper?” only a JC would say “no”. We can also ask “are you staff,” and only a JC would say “yes.”
safibn 2021-05-19 19:09:08
Part (c) Don and Viv are having a conversation as you pass by. Is there a single statement you can overhear from that conversation from which you can deduce that Don and Viv are both in the same group of people (both campers, or both JCs, or both mentors) without giving you any additional information about which group it is?
safibn 2021-05-19 19:09:37
We’d like to find a single statement that essentially says: “we are in the same group”. What distinguishes people in the same group?
Arnuv1000 2021-05-19 19:10:04
they trust each other
olive0827 2021-05-19 19:10:04
do not lie to each other
pi271828 2021-05-19 19:10:04
They wont lie to each other
juliankuang 2021-05-19 19:10:04
they both tell truth to each other
gordonhero 2021-05-19 19:10:04
They tell the truth both ways
GreenFrog1 2021-05-19 19:10:04
They bot tell the truth.
SHZhang 2021-05-19 19:10:04
The two people tell the truth to each other
m0bius 2021-05-19 19:10:04
they tell the truth to each other
bronzetruck2016 2021-05-19 19:10:04
They don't lie to each other.
student20201011 2021-05-19 19:10:04
they tell each other the truth
w_aops 2021-05-19 19:10:04
They tell the truth to each other
CoolCarsOnTheRun 2021-05-19 19:10:13
there's no lying between the two of them
llddmmtt1 2021-05-19 19:10:13
they don't lie to eachother
Edelweiss_USA 2021-05-19 19:10:13
They don't lie to each other.
Batee5a 2021-05-19 19:10:13
they'll both say the truth
ypang9804 2021-05-19 19:10:13
They both will lie to a certain group of people
pritiks 2021-05-19 19:10:13
they say the truth to eachother
GreenFrog1 2021-05-19 19:10:13
They both tell the truth.
safibn 2021-05-19 19:10:16
Two people are in the same group if and only if they both tell the truth to each other. Can anyone in a lying pair say that both of them tell the truth to each other?
safibn 2021-05-19 19:10:49
A lot of people have the right idea here!
safibn 2021-05-19 19:11:01
Without loss of generality, assume that Viv says: “you are not lying to me.” If Viv was lying, then Don must be lying. However, by the circle of lies, Don must be telling the truth, a contradiction.
safibn 2021-05-19 19:11:12
If Viv was telling the truth, then Don must also be telling the truth.
safibn 2021-05-19 19:11:22
So if we overhear “you are not lying to me,” we know that Viv and Don are in the same group.
safibn 2021-05-19 19:11:33
A super-short solution can be: “I agree” because if we overhear that, it’s equivalent to overhearing “you are not lying to me.”
safibn 2021-05-19 19:11:50
Thanks for coming by! Enjoy problem 2!
KevinCarde 2021-05-19 19:12:07
Hi all, I'm Kevin, and I'll be presenting Problem 2, which involves a lot of probability!
KevinCarde 2021-05-19 19:12:18
Since the full problem with all of its parts is quite long, we'll keep only parts of the problem stickied at any given time. So let's dive in:
KevinCarde 2021-05-19 19:12:25
Problem 2 Statement:
KevinCarde 2021-05-19 19:12:33
Kai has a collection of magic, shape-changing dice. When he rolls a blue die, it changes shape to have a number of sides equal to the number rolled minus one. For example, if a blue die starts out with 12 sides, then Kai has an equal probability of rolling any of the twelve integers from 1 through 12. If he rolls an 8, then the blue die changes shape to have 7 sides, and the next roll will have an equal probability of rolling any of the seven integers from 1 through 7. After rolling the same blue die over and over, eventually Kai will roll a 1, at which point the die will disappear.
KevinCarde 2021-05-19 19:12:45
Part a: Suppose the blue die starts with 4 sides (and so it has an equal probability of rolling any of the four integers from 1 through 4). How many rolls, on average, will it take for the die to disappear?
KevinCarde 2021-05-19 19:13:06
Part a (and b, coming up soon), are warm-ups with explicit numbers rather than variables, though they can still be tricky if probability is somewhat new to you, and then c, d, and e will involve proofs.
CircleInvert 2021-05-19 19:13:12
Time for some classic expected value
KevinCarde 2021-05-19 19:13:33
Indeed! "Expected value" is a fancier way of saying "average" for our purposes, since we don't necessarily expect people working on the quiz to have a background in probability
KevinCarde 2021-05-19 19:13:49
The pace that we move through this problem will accelerate as we go through all the parts, but the transcript will be available online shortly after this Math Jam if you'd like more time to puzzle over some of the trickier details.
KevinCarde 2021-05-19 19:14:11
Here in Part a, Kai's rolling a 4-sided die, which means there are four outcomes: he rolls 1, 2, 3, or 4. If he rolls a 1, then we're done after 1 roll; for any other roll, we'll need to know how many rolls it takes for the die to disappear starting from 1, 2, or 3 sides. So let's build this up step by step!
KevinCarde 2021-05-19 19:14:31
If Kai rolls a 1-sided die: The die always disappears after 1 roll, so the average number of rolls with a 1-sided die is 1.
KevinCarde 2021-05-19 19:14:41
What about a 2-sided die? How many rolls on average before it disappears?
KevinCarde 2021-05-19 19:15:11
I'm seeing a bunch of answers between 1 and 2 -- 1 and 2 are both possibilities, so the average number should be between them.
sj2015 2021-05-19 19:15:18
1.5
bronzetruck2016 2021-05-19 19:15:18
$\frac{3}{2}$
physicsbaby 2021-05-19 19:15:18
1.5
SHZhang 2021-05-19 19:15:18
$\frac{3}{2}$
MirmanStudent08 2021-05-19 19:15:18
1.5
pritiks 2021-05-19 19:15:18
3/2
kfcruan 2021-05-19 19:15:18
1.5
student20201011 2021-05-19 19:15:18
1.5
KevinCarde 2021-05-19 19:15:29
We'll write this out carefully:
KevinCarde 2021-05-19 19:15:53
2-sided die: There's a 50% chance to roll a 1, meaning the die disappears after 1 roll. This contributes 1/2*1 to the average number of rolls. There's a 50% chance to roll a 2, meaning the die becomes 1-sided, and it disappears after one more roll. This contributes 1/2*(1+1) to the average. In total, then, the average number of rolls is 3/2.
KevinCarde 2021-05-19 19:16:11
What about a 3-sided die?
KevinCarde 2021-05-19 19:16:32
(I might not give you quite enough time to compute this from scratch, but hopefully you have a chance to think about it a bit!)
KevinCarde 2021-05-19 19:16:50
I'm seeing a lot of 2s -- and it's very close to 2, but not quite.
MirmanStudent08 2021-05-19 19:17:05
11/6
w_aops 2021-05-19 19:17:05
1 + (0 + 1 + 3/2)/3
SHZhang 2021-05-19 19:17:05
\frac{11}{6}
SHZhang 2021-05-19 19:17:05
$\frac{11}{6}$
KevinCarde 2021-05-19 19:17:11
3-sided die: Let's list this out quickly:
KevinCarde 2021-05-19 19:17:17
* 1/3 chance to roll a 1, and the die disappears. This contributes 1/3*1 to the average.
KevinCarde 2021-05-19 19:17:27
* 1/3 chance to roll a 2, and the die becomes 1-sided, which we've computed. This contributes 1/3*(1+1).
KevinCarde 2021-05-19 19:17:40
* 1/3 chance to roll a 3, and the die becomes 2-sided, which we've computed. This contributes 1/3*(1+3/2), where the 3/2 is the average number of rolls for a 2-sided die to disappear.
KevinCarde 2021-05-19 19:17:57
Adding up all these contributions, we get that the average number of rolls is 11/6.
KevinCarde 2021-05-19 19:18:15
(The way w_aops wrote it out is very nice: it was the first roll, then a 1/3 chance each of 0, 1, or 3/2 further rolls!)
KevinCarde 2021-05-19 19:18:38
Okay, now let's actually do part 4. In the interest of time, I'll write it out quickly:
KevinCarde 2021-05-19 19:18:48
(Though some of you are beating me to it!)
bronzetruck2016 2021-05-19 19:18:52
$\frac{25}{12}$
MirmanStudent08 2021-05-19 19:18:52
25/12
KevinCarde 2021-05-19 19:19:00
4-sided die: Computing as before, rolling a 1 contributes 1/4*1, rolling a 2 contributes 1/4*(1+1), rolling a 3 contributes 1/4*(1+3/2), and rolling a 4 contributes 1/4*(1+11/6). Adding them up, we get a final answer of 25/12 rolls.
gordonhero 2021-05-19 19:19:09
1+(0+1+3/2+11/6)/4
KevinCarde 2021-05-19 19:19:23
This is very similar to how we wrote it out, and the analog of what w_aops wrote for 3-sided dice!
KevinCarde 2021-05-19 19:19:47
If you play with these numbers a bit, you might discover that $25/12 = 1 + 1/2 + 1/3 + 1/4$. Sums of reciprocals like this are called harmonic numbers, and there are several ways to prove that the $N$th harmonic number gives the average number of rolls before an $N$-sided die disappears. Play around with this more later if it seems interesting, but let's keep moving for now!
KevinCarde 2021-05-19 19:20:25
So, that's one of our warmups in Part a; now we're going to have Kai start playing games with Helena.
KevinCarde 2021-05-19 19:20:33
Now Kai is playing a game with Helena. Kai rolls a blue die first (causing it to change shape), then Helena rolls the same blue die (which changes shape again), and then they continue to take turns rolling the same die until one of them rolls a 1. Whoever rolls the 1 loses.
Part b: Suppose the blue die starts with 4 sides. What is the probability that Kai wins?
Part c: Suppose the blue die starts with $N$ sides. What is the probability that Kai wins?
KevinCarde 2021-05-19 19:20:50
Let's define $p(N)$ to be the probability that the first player wins the game with an $N$-sided die.
KevinCarde 2021-05-19 19:21:05
Note that I said "first player," not "Kai," because if Kai rolls, say, a 4 on his first roll, then Helena now is acting as if she's the first player in a game with a 3-sided die, so Helena wins with probability $p(3)$ from here, and hence Kai wins this specific case with probability $1 - p(3)$.
KevinCarde 2021-05-19 19:21:38
Okay, let's now try to compute $p(4)$, which is Part b. Like with Part a, let's build this up from smaller values:
KevinCarde 2021-05-19 19:21:44
What's $p(1)$? In other words, what's the chance Kai wins a game with a 1-sided die?
mathy88 2021-05-19 19:22:07
0
peace09 2021-05-19 19:22:07
0!
bronzetruck2016 2021-05-19 19:22:07
0
gordonhero 2021-05-19 19:22:07
0% I mean
kxiang 2021-05-19 19:22:07
0
sj2015 2021-05-19 19:22:07
0
SHZhang 2021-05-19 19:22:07
0
KevinCarde 2021-05-19 19:22:17
(I'm assuming the "0!" is enthusiasm, not factorial )
KevinCarde 2021-05-19 19:22:30
And yes:

* $p(1) = 0$, as Kai is guaranteed to lose immediately with a 1-sided die.
KevinCarde 2021-05-19 19:22:43
What about $p(2)$?
w_aops 2021-05-19 19:23:05
1/2
gordonhero 2021-05-19 19:23:05
50%
bronzetruck2016 2021-05-19 19:23:05
$\frac{1}{2}$
SHZhang 2021-05-19 19:23:05
$\frac{1}{2}$
peace09 2021-05-19 19:23:05
$\tfrac{1}{2}$
student20201011 2021-05-19 19:23:05
50
sj2015 2021-05-19 19:23:05
1/2
mathy88 2021-05-19 19:23:17
1/2 either Kai rolls a 1 or gives Helena P(1)
KevinCarde 2021-05-19 19:23:23
Great explanation!
KevinCarde 2021-05-19 19:23:27
* $p(2) = 1/2$, as there's a 50% chance Kai rolls a 1 and loses immediately, and a 50% chance Kai rolls a 2 and hands Helena a 1-sided die, causing her to lose immediately.
KevinCarde 2021-05-19 19:23:41
$p(3)$? (getting trickier...)
KevinCarde 2021-05-19 19:24:27
I'm seeing some interesting guesses: remember that even if Kai doesn't immediately lose (by rolling a 1), that doesn't necessarily mean he wins
gordonhero 2021-05-19 19:24:34
1/2
kxiang 2021-05-19 19:24:34
1/2?
bronzetruck2016 2021-05-19 19:24:34
Also $\frac{1}{2}$
Lilavigne 2021-05-19 19:24:34
1/2 again
KevinCarde 2021-05-19 19:24:47
I'll write out the full logic:
KevinCarde 2021-05-19 19:24:58
* For $p(3)$, there's a 1/3 chance of rolling a 3, handing Helena a 2-sided die, where we already know each player has a 1/2 chance of winning. Then there's a 1/3 chance of rolling a 2, causing Helena to lose instantly, and a 1/3 chance of rolling a 1, causing Kai to lose instantly. Thus $p(3) = 1/3*(1/2 + 1 + 0) = 1/2$.
Batee5a 2021-05-19 19:25:06
$1/3*(0+1/2+1)$
mc21s 2021-05-19 19:25:06
1/3*0+1/3*1+1/3*1/2
bronzetruck2016 2021-05-19 19:25:15
You may not send the same message twice in a row
KevinCarde 2021-05-19 19:25:27
Uh oh, that might be an issue with this problem
apricot_sherry 2021-05-19 19:25:35
is it the case that it is 1/2 with any die?
KevinCarde 2021-05-19 19:25:42
Hmm, things are getting suspicious, aren't they?
KevinCarde 2021-05-19 19:25:53
Indeed, we can do a similar calculation for $p(4)$, and we'll get 1/2
mc21s 2021-05-19 19:25:58
1/4(0+1+1/2+1/2)
Edelweiss_USA 2021-05-19 19:26:07
It's always(except for 1) $\frac{1}{2}$.
gordonhero 2021-05-19 19:26:07
$\frac{1}{2}$ wao fun game
KevinCarde 2021-05-19 19:26:13
Fair games are fun! Maybe.
KevinCarde 2021-05-19 19:26:32
So we're starting to suspect that, other than $N=1$, it's always a completely fair game: 1/2 chance for Kai to win, 1/2 for Helena.
w_aops 2021-05-19 19:26:34
Induction to prove for all die?
KevinCarde 2021-05-19 19:26:51
This was the most popular strategy in the applications, but we'll do some induction later, so we're going to present a different argument.
KevinCarde 2021-05-19 19:27:21
Often in math, if you want to prove that two sets have the same size, you can do this by constructing a bijection between them.
Arcticturn 2021-05-19 19:27:48
what is a bijection
rickiebosun 2021-05-19 19:27:48
. What's a bijection
KevinCarde 2021-05-19 19:28:03
Excellent question!
mc21s 2021-05-19 19:28:05
1-1 correspondence
KevinCarde 2021-05-19 19:28:34
Yes: a bijection is a way to have a 1 to 1 correspondence between the two sets, so that everything in each set corresponds to exactly one object in the other.
KevinCarde 2021-05-19 19:28:52
When working with probability, we can similarly use a bijection to prove that something happens 1/2 the time, but we have to be careful: we have to make sure that our bijection always takes one successful outcome to an equally likely unsuccessful outcome.
KevinCarde 2021-05-19 19:29:11
So, let's construct a bijection between possible sequences of die rolls where Kai wins and ones where Helena wins, making sure that our bijection preserves probabilities. Any ideas?
KevinCarde 2021-05-19 19:30:07
So I'm seeing some ideas about rolling a 1 vs rolling a 2, and that's a key idea here.
gordonhero 2021-05-19 19:30:11
rolling 1 vs rolling a 2
KevinCarde 2021-05-19 19:30:20
That's exactly what I just said
KevinCarde 2021-05-19 19:30:27
(But gordonhero did say it just before me, I promise)
KevinCarde 2021-05-19 19:30:48
If, say, Kai rolls a 2 at any point, then Helena automatically loses. However, Kai is just as likely to roll a 1 at that point instead of a 2, causing him to lose.
KevinCarde 2021-05-19 19:30:59
This gives us a pair of equally likely games, one of which Kai wins, and one of which Helena wins.
KevinCarde 2021-05-19 19:31:13
Similarly, if Helena rolls a 2, this is just as likely as her rolling a 1 at that point, and we get a similar pair of equally likely games.
KevinCarde 2021-05-19 19:31:32
So, that's our bijection: every game that Kai wins is in bijection to an equally likely game where Helena wins, simply by inserting or removing a roll of 2.
KevinCarde 2021-05-19 19:31:50
The existence of this bijection immediately proves that $p(N) = 1/2$ for $N \ge 2$!
KevinCarde 2021-05-19 19:32:13
This kind of argument can be a little tricky if you haven't seen something like it before, so I encourage you to think about it more if you need to.
Batee5a 2021-05-19 19:32:47
and if they roll another number n it can simply be considered as a new game with n-1 sides?
KevinCarde 2021-05-19 19:33:21
So we're thinking about entire games here: for example, if Kai rolls 5, then Helena 3, then Kai 1, that sequence of rolls has exactly the same probability of occurring as if Kai rolled 5, then Helena 3, then Kai 2, then Helena 1, but now the winner has swapped.
KevinCarde 2021-05-19 19:33:56
You may have some pondering to do, but since we have a lot to cover, we're going to move onwards to some fancier green dice!
KevinCarde 2021-05-19 19:34:05
Kai also has green dice, which work slightly differently from blue dice. When he rolls a green die, it changes shape to have a number of sides equal to the number rolled (so if Kai rolls the maximum possible value, the green die doesn't change shape at all).
KevinCarde 2021-05-19 19:34:14
Part d: He plays the same game with Helena, but now with an $N$-sided green die; the first player to roll a 1 still loses. What is the probability that Kai wins?
KevinCarde 2021-05-19 19:34:34
So: green dice are very similar to blue dice, but now the number of sides doesn't automatically decrease.
KevinCarde 2021-05-19 19:34:55
It's similar to the last game, so we might expect the probability to be close to 1/2, but it's unclear if it'll be exactly 1/2, like it was before.
KevinCarde 2021-05-19 19:35:07
So let's look for a pattern!
KevinCarde 2021-05-19 19:35:20
Keeping $p(N)$ as the probability that the first player wins, what's $p(2)$?
KevinCarde 2021-05-19 19:35:25
(Already this is a little trickier to compute!)
bronzetruck2016 2021-05-19 19:35:59
$\frac{1}{3}$
apricot_sherry 2021-05-19 19:36:06
1/3
Arcticturn 2021-05-19 19:36:06
1/3
KevinCarde 2021-05-19 19:36:17
We can think of it this way: Kai has a 50% chance of losing immediately, and a 50% chance of handing Helena a 2-sided die, where Helena has by definition a $p(2)$ chance of winning, and thus Kai has a $1-p(2)$ chance of winning.
KevinCarde 2021-05-19 19:36:26
Putting it together, $p(2) = 1/2*(0 + 1-p(2))$, or $3/2 p(2) = 1/2$, or $p(2) = 1/3$.
KevinCarde 2021-05-19 19:36:40
(Sorry for the ugly latex *, my bad!)
KevinCarde 2021-05-19 19:36:55
In the interest of time, we won't explicitly compute more values, but we'd get $p(3) = 5/12$ and $p(4) = 9/20$. Notice any pattern?
MirmanStudent08 2021-05-19 19:37:22
All slightly less than 1/2
awesomeguy856 2021-05-19 19:37:22
denominator is n(n-1)
kxiang 2021-05-19 19:37:22
Close to $\frac{1}{2}$
fuwa 2021-05-19 19:37:29
always a unit smaller than a half?
bronzetruck2016 2021-05-19 19:37:34
Closer and closer to 2. Maybe $\frac{1}{2}-\frac{1}{(n)(n+1)}$?
KevinCarde 2021-05-19 19:37:39
Aha! An explicit conjecture!
KevinCarde 2021-05-19 19:37:52
And indeed, bronzetruck2016's formula works with all our examples so far.
KevinCarde 2021-05-19 19:38:24
Let's think about Kai's first roll based on whether he rolls $N$ or not:
apricot_sherry 2021-05-19 19:38:54
isn't it similar to the previous problem with an extra case?
KevinCarde 2021-05-19 19:39:05
It is, but that extra case -- where we can repeat the number that was just rolled -- makes it pretty tricky!
KevinCarde 2021-05-19 19:39:09
Okay, back to Kai's first roll:
KevinCarde 2021-05-19 19:39:17
* Kai rolls $N$, which occurs $1/N$ of the time: We're now playing exactly the same game (since the die still has $N$ sides), but now Helena goes first, so Kai wins with probability $(1 - p(N))$.
KevinCarde 2021-05-19 19:39:41
* Kai rolls something else, which occurs $(N-1)/N$ of the time: In this case, Kai is equally likely to roll each of the numbers 1 through $N-1$. But we have notation for Kai's win chance when he's equally likely to roll $1$ through $N-1$: it's simply $p(N-1)$. So $p(N-1)$ is the chance that Kai wins in this case.
KevinCarde 2021-05-19 19:39:54
Putting these together, the total probability that Kai wins is
KevinCarde 2021-05-19 19:40:01
$$

p(N) = \frac{1}{N} (1 - p(N)) + \frac{N-1}{N} p(N-1)

$$
KevinCarde 2021-05-19 19:40:19
This is great, as we can now solve for $p(N)$ in terms of $p(N-1)$!
KevinCarde 2021-05-19 19:40:26
$$

p(N) = \frac{1}{N+1} + \frac{N-1}{N+1} p(N-1)

$$
KevinCarde 2021-05-19 19:41:28
Recurrence relations like this are an excellent place to use induction to prove that it works for all possible values of $N$. If you're unfamiliar with induction, we won't be able to do a full introduction here, but the strategy is to prove a "base case" (generally the smallest value of $N$ we need to show), then prove that if it works for $N-1$, it works for $N$.
KevinCarde 2021-05-19 19:41:39
What's our base case?
gordonhero 2021-05-19 19:42:05
N=1
bronzetruck2016 2021-05-19 19:42:05
$N=1$
physicsbaby 2021-05-19 19:42:05
n=2
KevinCarde 2021-05-19 19:42:43
Either $N=1$ or $N=2$ is fine! (As long as you've shown it for both $N=1$ and $N=2$, it doesn't really matter which you choose as your base case.)
KevinCarde 2021-05-19 19:43:15
for $N=1$, we have $p(1) = 0$ (since Kai immediately loses), which does agree with bronzetruck2016's conjectured formula (which I've pinned)
KevinCarde 2021-05-19 19:43:28
for $N=2$, we computed $p(2) = 1/3$, which also agrees with the formula.
KevinCarde 2021-05-19 19:43:33
Now we must carry out the inductive step: assuming our conjecture is true for $p(N-1)$, show that it's true for $p(N)$.
KevinCarde 2021-05-19 19:44:18
So let's plug in $p(N-1) = \frac{1}{2} - \frac{1}{(N-1)*N}$ into our recurrence!
KevinCarde 2021-05-19 19:44:38
$$

p(N) = \frac{1}{N+1} + \frac{N-1}{N+1} \left(\frac{1}{2} - \frac{1}{(N-1)N}\right)

$$
llddmmtt1 2021-05-19 19:44:47
why $*$
KevinCarde 2021-05-19 19:44:54
I keep being bad at latex -- I just mean multiplication
KevinCarde 2021-05-19 19:45:14
We've plugged in $p(N-1)$ and now we can simplify:
KevinCarde 2021-05-19 19:45:16
$$

p(N) = \frac{N+1}{2(N+1)} - \frac{1}{(N+1)N} = \frac{1}{2} - \frac{1}{N(N+1)}

$$
KevinCarde 2021-05-19 19:45:26
And that's exactly the formula we wanted to compute for $p(N)$, which completes the inductive step and the proof!
KevinCarde 2021-05-19 19:45:58
Okay! If induction is unfamiliar to you, that might have been fast and/or mysterious. But we'll skip induction for this last part
KevinCarde 2021-05-19 19:46:04
Part e: Suppose now that Kai and Helena modify their game: the first person to roll one of the numbers 1, 2, …, $k$ loses. If they start with an $N$-sided green die, what is the probability that Kai wins?
gordonhero 2021-05-19 19:46:33
isn't the base case k now..???
KevinCarde 2021-05-19 19:46:41
Yes! That is the key difference in how the math will play out.
KevinCarde 2021-05-19 19:46:50
If $p(N)$ is Kai's probability of winning in this new game, the exact same logic that led to our recurrence relation still holds as long as $N > k$, and so the exact same recurrence relation holds for $N > k$!
KevinCarde 2021-05-19 19:47:01
So indeed, the recurrence relation is the same, but our starting point is now $N = k$, not $N=1$.
KevinCarde 2021-05-19 19:47:12
So since the smallest instance of our recurrence relation writes $p(k+1)$ in terms of $p(k)$, we'll need to know $p(k)$ to get started. What is it?
llddmmtt1 2021-05-19 19:47:45
0?
mayasu 2021-05-19 19:47:45
0
gordonhero 2021-05-19 19:47:45
0%
Batee5a 2021-05-19 19:47:45
0
physicsbaby 2021-05-19 19:47:45
0
KevinCarde 2021-05-19 19:48:01
Right: in this new game, any roll from 1 through $k$ is an instant loss, so Kai cannot survive the first roll of a $k$-sided die!
KevinCarde 2021-05-19 19:48:26
Starting with $p(k) = 0$ and using the recurrence for $p(k+1)$, we see $p(k+1) = 1/(k+2)$, and it gets messier from there. With enough experimentation, we could come up with a correct conjecture, but we're going to use a bit of a trick to simplify our recurrence.
KevinCarde 2021-05-19 19:48:56
In Part d, the answer turned out to be that Kai wins $1/(N(N+1))$ less than a fair game. Inspired by this, let's define $q(N) = 1/2 - p(N)$, so that $q(N)$ measures how far from fair the game is. We can rewrite our recurrence
KevinCarde 2021-05-19 19:49:03
$$

p(N) = \frac{1}{N} (1 - p(N)) + \frac{N-1}{N} p(N-1)

$$
KevinCarde 2021-05-19 19:49:10
by substituting $p(N) = 1/2 - q(N)$, and we hope it'll look nicer:
KevinCarde 2021-05-19 19:49:20
$$

1/2 - q(N) = \frac{1}{N} (1/2 + q(N)) + \frac{N-1}{N} (1/2 - q(N-1))

$$
KevinCarde 2021-05-19 19:49:34
And indeed, after simplifying:

$$

q(N) = \frac{N-1}{N+1} q(N-1)

$$
KevinCarde 2021-05-19 19:50:02
This is fantastic, as we can build up values of $q$ via a telescoping product! Working our way from $q(N)$ down to $q(k) = 1/2$ (corresponding to $p(k) = 0$ that we talked about earlier),
KevinCarde 2021-05-19 19:50:27
(oops, let's try that again)
KevinCarde 2021-05-19 19:50:35
$$

q(N) = \frac{(N-1)(N-2) \dots (k+1)k}{(N+1)(N) \dots (k+3)(k+2)} q(k)

$$
KevinCarde 2021-05-19 19:50:44
(whew! better this time )
KevinCarde 2021-05-19 19:51:08
This is the really fun part where you get to cancel (almost) everything!
KevinCarde 2021-05-19 19:51:21
And plugging in $q(k) = 1/2$, we end up with

$$

q(N) = \frac{k(k+1)}{N(N+1)} \frac{1}{2}.

$$
KevinCarde 2021-05-19 19:51:36
Since Kai's win probability is $p(N) = 1/2 - q(N)$, we have now shown that this is
KevinCarde 2021-05-19 19:51:48
$$

p(N) = \frac{1}{2} - \frac{k(k+1)}{2N(N+1)}

$$
KevinCarde 2021-05-19 19:51:52
and we're done!!
KevinCarde 2021-05-19 19:52:05
No induction needed, because our recurrence in terms of $q$ was so nice
dan09 2021-05-19 19:52:25
So it is $\frac{k(k+1)}{2N(N+1)}$ from a fair game?
KevinCarde 2021-05-19 19:52:33
Indeed! So it becomes very close to fair as $N$ gets very big
peace09 2021-05-19 19:52:36
$P(N)=\tfrac{T_n-T_k}{2T_n},$ which is pretty interesting.
KevinCarde 2021-05-19 19:52:45
Yes! There are a lot of interesting things to think more about
KevinCarde 2021-05-19 19:53:01
(There are some really nice ways to go through all this in terms of generating functions, if you've seen those before!)
KevinCarde 2021-05-19 19:53:22
We've had to go quickly, but the transcript will be available online for you to go through the details at a more reasonable pace.
KevinCarde 2021-05-19 19:53:23
But since we have a very finite amount of time tonight, let's pass things on to Sara for Problem 3!
SMF 2021-05-19 19:53:34
Hi everyone! I’m Sara and I’ll be presenting Problem 3. Here is the problem statement:
SMF 2021-05-19 19:53:44
Problem 3: A construction company is designing a new apartment complex. They have an $n \times n$ lot in which each $ 1 \times 1 $ square can either occupy an apartment or a garden.
SMF 2021-05-19 19:53:48
The apartments must all have a scenic view: if an apartment is not on the edge of the apartment complex, then one of the $4$ adjacent lots must have a garden.
SMF 2021-05-19 19:53:52
For reference, consider the first two diagrams below: diagram (i) shows a valid design, while the design in diagram (ii) has one apartment without a scenic view.
SMF 2021-05-19 19:53:58
https://www.mathcamp.org/files/yearly/2021/quiz/qq_diagram_i.png
https://www.mathcamp.org/files/yearly/2021/quiz/qq_diagram_ii.png
SMF 2021-05-19 19:54:17
Problem 3a: What is the minimal number of gardens if $n = 6$?
dan09 2021-05-19 19:54:27
Oh I remember this question, it was my favorite one
SMF 2021-05-19 19:54:38
Mine too! That's why I'm happy to be presenting it
SMF 2021-05-19 19:55:02
I'm seeing some correct answers already—let's go through the solution together.
SMF 2021-05-19 19:55:07
Let's draw a $6 \times 6$ grid:
SMF 2021-05-19 19:55:11
https://www.mathcamp.org/static/yearly/2021/mathjams/3a_1
SMF 2021-05-19 19:55:17
Which squares automatically have scenic views?
kxiang 2021-05-19 19:55:36
outside edge
kfcruan 2021-05-19 19:55:36
the sides
CoolCarsOnTheRun 2021-05-19 19:55:36
ones on the border
Lilavigne 2021-05-19 19:55:36
the border
m0bius 2021-05-19 19:55:36
the outer squares
w_aops 2021-05-19 19:55:36
the ones on the border
m0bius 2021-05-19 19:55:36
the outline
SMF 2021-05-19 19:55:41
Right! The squares on the outer edge already have scenic views, so we may as well place happy apartments there:
SMF 2021-05-19 19:55:45
https://www.mathcamp.org/static/yearly/2021/mathjams/3a_2
SMF 2021-05-19 19:55:54
How many gardens do we need to satisfy the apartments in the inner $4 \times 4$ square? (Some of you beat me to this question.)
MirmanStudent08 2021-05-19 19:56:16
4
kxiang 2021-05-19 19:56:16
4
CircleInvert 2021-05-19 19:56:16
4
gxxk 2021-05-19 19:56:16
4
Arcticturn 2021-05-19 19:56:16
Center 4
bronzetruck2016 2021-05-19 19:56:16
4
physicsbaby 2021-05-19 19:56:16
4
juliankuang 2021-05-19 19:56:16
4
Lilavigne 2021-05-19 19:56:16
4
CoolCarsOnTheRun 2021-05-19 19:56:16
4
w_aops 2021-05-19 19:56:16
4
truffle 2021-05-19 19:56:16
4
SMF 2021-05-19 19:56:26
Yep! Two ways we can do this are:
SMF 2021-05-19 19:56:33
https://www.mathcamp.org/static/yearly/2021/mathjams/3a_3
https://www.mathcamp.org/static/yearly/2021/mathjams/3a_4
SMF 2021-05-19 19:56:37
(For an extra challenge, think about why these are the only two ways to minimally satisfy a $4 \times 4$ grid.)
SMF 2021-05-19 19:56:45
So, we've shown that the minimum is at most $4$. A common mistake students made was not showing that the minimum is exactly $4$. Specifically—how can we be sure that $4$ gardens are required? Why can't we satisfy all apartments with $3$ gardens?
peace09 2021-05-19 19:56:53
One for each of the corners!
SMF 2021-05-19 19:56:56
One way to show this is by looking at the corners of the $4 \times 4$ square.
SMF 2021-05-19 19:57:01
https://www.mathcamp.org/static/yearly/2021/mathjams/3a_5
SMF 2021-05-19 19:57:07
Can one garden satisfy more than one corner?
juliankuang 2021-05-19 19:57:21
no
bronzetruck2016 2021-05-19 19:57:21
noo
SHZhang 2021-05-19 19:57:21
No!
m0bius 2021-05-19 19:57:21
No
truffle 2021-05-19 19:57:21
No
fancyamazon 2021-05-19 19:57:21
no
Lilavigne 2021-05-19 19:57:21
no
SMF 2021-05-19 19:57:26
Right! No garden can satisfy more than one corner, because any two corners have at least two squares between them, and any two squares satisfied by a garden have at most one square between them.
SMF 2021-05-19 19:57:31
//cdn.artofproblemsolving.com/images/4/d/e/4de88565c8159be229c5864860d92fd4212cc60d.png
SMF 2021-05-19 19:57:36
So, we've proven that the minimal number of gardens needed for a $6 \times 6$ apartment complex is $4$.
CircleInvert 2021-05-19 19:57:39
Do part B first and apply it to part A
SMF 2021-05-19 19:57:42
Another way to show it is to just use part (b), which we'll prove now.
SMF 2021-05-19 19:57:47
Problem 3b: Prove that the number of gardens must be at least $\frac{1}{5} (n-2)^2$.
SMF 2021-05-19 19:58:08
We can start by drawing a $n \times n$ grid:
SMF 2021-05-19 19:58:13
https://www.mathcamp.org/static/yearly/2021/mathjams/3b_1
SMF 2021-05-19 19:58:19
Let's start by looking at the $(n-2)^2$ term. What might it represent in terms of this problem?
Lilavigne 2021-05-19 19:58:41
the inner square
peace09 2021-05-19 19:58:41
The central $n-2$ by $n-2$ grid.
conantwiz2023 2021-05-19 19:58:41
the number of non-boundary spaces to be covered
fancyamazon 2021-05-19 19:58:41
the squares not on the borders
CoolCarsOnTheRun 2021-05-19 19:58:41
the number of squares in the non-border lots
pritiks 2021-05-19 19:58:41
all the squares without the bordering squares
mathy88 2021-05-19 19:58:41
The squares excluding the borders
SMF 2021-05-19 19:58:48
Right. As in part (a), the outer squares already have a scenic view. So it suffices to place gardens to satisfy the inner squares—and there are $(n-2)^2$ of them.
SMF 2021-05-19 19:58:53
https://www.mathcamp.org/static/yearly/2021/mathjams/3b_2
SMF 2021-05-19 19:58:57
Now, where could the $\frac{1}{5}$ come from?
CoolCarsOnTheRun 2021-05-19 19:59:31
each garden makes at most 5 squares happy
Lilavigne 2021-05-19 19:59:31
the center of a "cross"
dan09 2021-05-19 19:59:31
There are multiple houses that get a scenic view from one garden
bronzetruck2016 2021-05-19 19:59:31
Each garden "takes care of" at most 5 squares.
m0bius 2021-05-19 19:59:31
4 spaces for 1 apartment 1/5
kxiang 2021-05-19 19:59:31
The number of lots a garden provides scenes to (5)
fancyamazon 2021-05-19 19:59:31
when you place a tree it can at most take care of itself and the edges
mathy88 2021-05-19 19:59:31
The garden satisfies 5 spaces, including the square it is in
SMF 2021-05-19 19:59:40
Yes. Each garden satisfies at most $4$ apartments. So, for every $4$ apartments we need at least $1$ garden. So at least $\frac{1}{5}$ of the squares in the inside must be gardens.
SMF 2021-05-19 19:59:45
This proves that any valid $n \times n$ apartment complex must have at least $\tfrac{1}{5} (n-2)^2$ gardens.
SMF 2021-05-19 19:59:52
On to part c!
SMF 2021-05-19 19:59:57
Problem 3c: Prove that it is possible to construct an apartment complex with no more than $\frac{1}{5} n^2$ gardens for any n.
SMF 2021-05-19 20:00:13
This question is asking for a construction. Since we just proved in part (b) that any construction must use at least $\frac{1}{5} (n-2)^2$ gardens, we will have to place the gardens efficiently in order to use only $\frac{1}{5} n^2$ of them.
SMF 2021-05-19 20:00:19
In particular, we want to avoid situations where a single apartment has a view of multiple gardens, since those gardens are being “wasted”.
SMF 2021-05-19 20:00:28
For example, here is an inefficient construction which uses approximately $\frac{1}{4} n^2$ gardens. The apartment highlighted green has a view of $2$ gardens, which is more than strictly necessary.
SMF 2021-05-19 20:00:33
https://www.mathcamp.org/static/yearly/2021/mathjams/3c_3
SMF 2021-05-19 20:00:51
So, how can we lay out the gardens efficiently?
kxiang 2021-05-19 20:01:35
tetrate the crosses.
CoolCarsOnTheRun 2021-05-19 20:01:35
tesselate the crosses that they affect
w_aops 2021-05-19 20:01:35
Connect them like jigsaw pieces
bronzetruck2016 2021-05-19 20:01:35
"knight's move away"
artistic_girl 2021-05-19 20:01:35
tesselate
SMF 2021-05-19 20:01:40
Great! We can think about a garden satisfying 4 apartments neighboring it:
SMF 2021-05-19 20:01:53
//cdn.artofproblemsolving.com/images/6/4/e/64ea4bbf6c4964e24ed267f382872559bd3946dc.png
SMF 2021-05-19 20:02:02
To design a valid apartment complex, all we need to do is place these + shapes so that no square is left uncovered. We can do this by tiling the + shapes in the following way:
SMF 2021-05-19 20:02:08
https://www.mathcamp.org/static/yearly/2021/mathjams/3c_5
SMF 2021-05-19 20:02:14
Awesome! This is super space efficient. Are we done yet?
bronzetruck2016 2021-05-19 20:02:36
no
dan09 2021-05-19 20:02:36
No
Lilavigne 2021-05-19 20:02:36
not yet
mathy88 2021-05-19 20:02:36
No, it isnt a square
kxiang 2021-05-19 20:02:36
No, some might go off the edge
peace09 2021-05-19 20:02:36
Not yet... this shape isn't a square!
RedFireTruck 2021-05-19 20:02:36
no
artistic_girl 2021-05-19 20:02:36
jagged edges
SMF 2021-05-19 20:02:50
Right! We want to place gardens and apartments into a $n \times n$ square, not a blocky blob. When we turn the above configuration into a square, we encounter two problems.
SMF 2021-05-19 20:02:59
https://www.mathcamp.org/static/yearly/2021/mathjams/3c_7
SMF 2021-05-19 20:03:08
(1) Some gardens end up outside the square. We need to be sure that the apartments they used to cover are still OK.
SMF 2021-05-19 20:03:14
(2) Some gardens are no longer covering $4$ apartments, so we can no longer guarantee the $\frac{1}{5}$ fraction.
SMF 2021-05-19 20:03:20
How do we deal with (1)?
physicsbaby 2021-05-19 20:04:07
ones they used to cover are on the sides so they are asfe
physicsbaby 2021-05-19 20:04:07
safe
Batee5a 2021-05-19 20:04:07
nothing, they're already on the edge so they have a view
m0bius 2021-05-19 20:04:07
It's okay because all the outer squares have a view
SMF 2021-05-19 20:04:19
Yep—we don’t have to do anything! Any garden which gets cut off would have only covered apartments on the border of the square. But, apartments on the border are already satisfied. Crisis averted!
SMF 2021-05-19 20:04:26
Dealing with (2) is a bit trickier. In our above example, when we took a $6 \times 6$ square chunk from the tiling, we ended up with an apartment complex with $8$ gardens. But $\frac{1}{5} 6^2 = 7.2 < 8,$ so we are using too many gardens.
SMF 2021-05-19 20:04:34
//cdn.artofproblemsolving.com/images/4/4/7/447972d4f6dd80243c775a99767648d75a5815ed.png
SMF 2021-05-19 20:04:43
So, we'll need to somehow adjust the tiling so that every apartment in our $n \times n$ grid is covered, and only $\frac{1}{5} n^2$ squares are used. How might we go about this?
SMF 2021-05-19 20:06:19
Lots of different ideas!
artistic_girl 2021-05-19 20:06:34
there is a pattern of the spilled over squares
SMF 2021-05-19 20:06:39
One way to solve this problem is to study the + tiling pattern carefully and look for patterns in how the inefficient red + signs "spill over".
SMF 2021-05-19 20:06:43
For each possible remainder of $n \mod 5$, a different pattern emerges. In each case, it can be shown that fewer than $\frac{1}{5} n^2$ gardens are needed.
SMF 2021-05-19 20:06:52
Some of you suggested a different approach which doesn't require casework:
wbender25 2021-05-19 20:07:02
We can translate the gardens and apply the pigeonhole principle.
kxiang 2021-05-19 20:07:02
shift it over 1
SMF 2021-05-19 20:07:11
Let's go back to the tiling we had of a $6 \times 6$ square.
SMF 2021-05-19 20:07:22
Label each part of the + sign with a $1,2,3,4,$ or $5$:
SMF 2021-05-19 20:07:27
//cdn.artofproblemsolving.com/images/2/f/0/2f048529a7d9c8730bc209a3a4de54c05d13a753.png
SMF 2021-05-19 20:07:33
Now back to the full picture:
SMF 2021-05-19 20:07:52
//cdn.artofproblemsolving.com/images/8/1/2/812f42ffbcd9f46ec641bb926f5b91e68d7156ef.png
SMF 2021-05-19 20:08:01
Notice that if we remove the green outlines, there is nothing special about the number $3.$ We can place gardens on any other number and they would satisfy all of the apartments in the grid.
SMF 2021-05-19 20:08:07
//cdn.artofproblemsolving.com/images/a/6/7/a67aa1b6a8f93a96416ea0d84ae5de428d9df829.png
SMF 2021-05-19 20:08:14
How can we make a construction with fewer than $\tfrac{1}{5} 6^2$ gardens from here?
w_aops 2021-05-19 20:09:33
5 as the center
mayasu 2021-05-19 20:09:33
the gardens should be on 4s
SHZhang 2021-05-19 20:09:33
Choose the least frequently occurring number and put gardens in squares with that number
SMF 2021-05-19 20:09:47
Right! Here, any number except $3$ appears $7$ times, which is less than $\tfrac{1}{5} 6^2 = 7.2.$ So, for example, we could place gardens on all of the $2$s, which would satisfy all apartments using only $7$ gardens.
SMF 2021-05-19 20:10:17
(Many of you mentioned that the problem with the previous arrangement of gardens was that we were wasting $4$ gardens on corners. Indeed, this problem goes away when we use a different number as the center of the + sign!)
SMF 2021-05-19 20:10:24
This works because of the way we numbered the squares—no matter which number we place the gardens on, all apartments except those on the boundary will be covered.
SMF 2021-05-19 20:10:41
//cdn.artofproblemsolving.com/images/f/a/d/fadb2b5f86e89d02c70fd548dc396b1a24f8573e.png
SMF 2021-05-19 20:11:22
Note that it's sometimes helpful to put a garden on the edge, even if it's not being used efficiently—that garden can cover apartments not on the edge.
SMF 2021-05-19 20:11:29
Now let's analyze the general $n \times n$ case. Let's label the squares of an $n \times n$ grid with the numbers $1, 2, 3, 4, 5$ in the same pattern.
SMF 2021-05-19 20:11:35
//cdn.artofproblemsolving.com/images/9/8/9/98957eb038d7394dcd6d4f45accf2590eabb111b.png
SMF 2021-05-19 20:11:41
Suppose there are $x_i$ squares labeled $i$ for $1 \leq i \leq 5.$ What is $x_1 + \dots + x_5$?
peace09 2021-05-19 20:12:27
$x_1 + x_2 + x_3 + x_4 + x_5 = n^2.$
bronzetruck2016 2021-05-19 20:12:27
$n^2$
kxiang 2021-05-19 20:12:27
$n^2$
iamhungry 2021-05-19 20:12:27
$n^2$
SMF 2021-05-19 20:12:32
Good, we have $x_1 + \dots + x_5 = n^2.$ Now, let $x_{min}$ be the smallest $x_1, ..., x_5$. What can we say about $x_{min}$?
robot317 2021-05-19 20:13:40
It is smaller than n^2/5
peace09 2021-05-19 20:13:40
By the Pigeonhole, the largest possible smallest $x_i$ is $\tfrac{n^2}{5}.$
SMF 2021-05-19 20:13:48
Yes! There will exist a number which has $\leq \frac15 n^2 $ squares, and we can place the gardens there.
SMF 2021-05-19 20:13:52
For example, if $2$ appears less than $\frac{1}{5} n^2$ times, then our construction would look like this:
SMF 2021-05-19 20:13:57
//cdn.artofproblemsolving.com/images/d/2/9/d298b2d0d0c17edf99f37551236a54d7ad386c2f.png
SMF 2021-05-19 20:14:04
We have a valid construction which uses $\leq \frac15 n^2 $ gardens. We're done with part (c)!
SMF 2021-05-19 20:14:16
Problem 3d: Now suppose that gardens have very tall trees that provide a scenic view to up to $20$ nearby lots. Specifically, if a garden is placed at the center of a $5 \times 5$ square, then every apartment in that square except the four $1 \times 1$ corners will have a scenic view of the garden.
SMF 2021-05-19 20:14:24
https://www.mathcamp.org/static/yearly/2021/mathjams/3d_1
SMF 2021-05-19 20:14:30
In other words, a single garden satisfies the following apartments:
SMF 2021-05-19 20:14:34
https://www.mathcamp.org/static/yearly/2021/mathjams/3d_2
SMF 2021-05-19 20:14:37
What upper and lower bounds can you prove on the number of gardens needed?
SMF 2021-05-19 20:15:00
So the question is asking for two things: an upper bound and a lower bound. Let's unpack that a little bit.
SMF 2021-05-19 20:15:05
What does it mean to prove that some number $L$ is a lower bound for this problem?
SwimWithDolphin 2021-05-19 20:15:50
not possible for there to be less than $L$ gardens
kxiang 2021-05-19 20:15:50
There will be at least $L$ gardens in any arrangement
dan09 2021-05-19 20:15:50
There is no setting that has less gardens than $L$, so $L$ is the most optimal setting of gardens
mayasu 2021-05-19 20:15:50
the least numer of gardebs needed
peace09 2021-05-19 20:15:54
We need at least $L$ gardens to make everyone happy!
SMF 2021-05-19 20:15:59
Right, you need to prove that at least $L$ gardens are required to make it work. This is like what we did in 3b, where we proved you need at least $\frac{1}{(n-2)^2}$ gardens to make it work.
SMF 2021-05-19 20:16:12
What does it mean to prove that some number $U$ is an upper bound?
kxiang 2021-05-19 20:17:16
In any case, there will be at least one arrangement that uses no more than $U$ gardens.
robot317 2021-05-19 20:17:16
We can place a max of U number of gardens to make everybody Happy
dan09 2021-05-19 20:17:16
We cannot possibly do worse than $U$, so $U$ is the least optimal setting of gardens
SMF 2021-05-19 20:17:26
Right, you need to show that it's possible to do it using only $U$ gardens. This is like what we did in 3c, where we showed that $\frac{1}{5} n^2$ is an upper bound by showing how to construct a pattern with that many gardens.
SMF 2021-05-19 20:17:44
So part 3d is asking you to do what you did in 3b and 3c, but for this more complicated situation.
SMF 2021-05-19 20:17:53
Let's start with the most straightforward lower bound, similar to 3b. How many gardens must we use at least, in order to give every apartment in a $n \times n$ grid a scenic view?
mathy88 2021-05-19 20:19:14
(n-2)^2/21
bronzetruck2016 2021-05-19 20:19:14
$\frac{1}{21}(n-2)^2$
SwimWithDolphin 2021-05-19 20:19:14
$\frac{(n-2)^2}{21}$?
SHZhang 2021-05-19 20:19:14
$\frac{1}{21}(n-2)^2$
Lilavigne 2021-05-19 20:19:14
1/21(n-2)^2
ingmom 2021-05-19 20:19:14
(1/21)(n-2)^2
SMF 2021-05-19 20:19:19
Excellent!
SMF 2021-05-19 20:19:33
The solution is similar to what we did in part (b)—try and apply similar ideas to get this bound.
SMF 2021-05-19 20:19:47
Now, let's take a quick look at the upper bound.
SMF 2021-05-19 20:20:17
To prove an upper bound on the minimum number of gardens needed, we need to provide an explicit construction using as few gardens as possible.
SMF 2021-05-19 20:20:23
What makes this problem tricky is that the shapes don't perfectly tile, unlike the + signs we worked with in part (c).
SMF 2021-05-19 20:20:30
//cdn.artofproblemsolving.com/images/0/3/7/03721cfec7a83a01ed45b97b07c8c4a398fc8204.png
SMF 2021-05-19 20:20:37
No matter how we place the shapes, if we try to get them not to overlap, "gaps" will occur (marked red in the above picture).
SMF 2021-05-19 20:20:49
Even if we can't make a perfectly efficient construction, we can get pretty close by letting the shapes slightly overlap:
SMF 2021-05-19 20:20:56
//cdn.artofproblemsolving.com/images/c/c/1/cc1d246027b01162a367653f93242d47d10928c1.png
SMF 2021-05-19 20:21:09
If we use this tiling to place gardens in an $n \times n$ grid, roughly how many gardens do we need? In other words, which upper bound does this idea give us?
SwimWithDolphin 2021-05-19 20:22:10
$\frac{n^2}{20}$
bronzetruck2016 2021-05-19 20:22:10
$\frac{n^2}{20}$
SMF 2021-05-19 20:22:22
Right! It's possible to do a lot of detail-oriented work to get this bound very close to $\frac{1}{20} n^2.$
SMF 2021-05-19 20:22:30
It turns out that the nice pigeonhole argument that we had in 3c doesn't quite work any more—I'll leave it as an exercise for you to figure out why. But you can use it to get a construction with $\frac{1}{20} n^2 + 4n - 12$.
SMF 2021-05-19 20:23:12
So let's recap where we are so far in Problem 3d.
SMF 2021-05-19 20:23:18
We showed that you need at least $\frac{1}{21}(n-2)^2$ gardens.
SMF 2021-05-19 20:23:25
We showed that you can definitely do it with $\frac{1}{20} n^2$ plus something linear in $n$.
SMF 2021-05-19 20:23:38
That's kind of unsatisfying! It's very different from 3b and 3c, where the leading term in both the upper and the lower bound was $\frac{1}{5}n^2$. So in that case, we basically figured out the true minimum number of gardens required, up to some lower order terms.
SMF 2021-05-19 20:23:45
But here, all we know is that the true minimum is somewhere between $\frac{n^2}{21}$ and $\frac{n^2}{20}$.
SMF 2021-05-19 20:23:57
There is no easy way to close the gap here, and before Mathcamp staff published this year's Quiz, we also had no clue what the exact growth rate of $(\text{minimum \# gardens})$ should be for this part.
SMF 2021-05-19 20:24:12
A few students went above and beyond in making progress toward an improved lower bound.
SMF 2021-05-19 20:24:15
Ethan from Vienna, VA improved the lower bound to roughly $\frac{n^2}{20.5}$.
SMF 2021-05-19 20:24:20
And Kevin from North Potomac, MD as well as Edwin from Cupertino, CA managed to improve the lower bound to roughly $\frac{n^2}{20}$, matching the upper bound up to lower-order terms!
SMF 2021-05-19 20:24:24
Their proofs use different techniques ranging from graph theory and convex geometry to show that there's no way to arrange the tiles perfectly so that each one satisfies more than $20.5$ or $20$ of the apartments.
SMF 2021-05-19 20:24:28
All three of these proofs are extremely impressive and creative! They're also quite complicated, so unfortunately, we don't have time to go through them now.
SMF 2021-05-19 20:25:25
That's it for this problem! If you enjoyed it, you might have fun playing around with different ways the garden can satisfy nearby apartments. For example:
SMF 2021-05-19 20:25:32
https://www.mathcamp.org/static/yearly/2021/mathjams/3e_1

https://www.mathcamp.org/static/yearly/2021/mathjams/3e_2
SMF 2021-05-19 20:25:38
And now I’ll pass it on to Laura, who’ll talk about the solution to Problem 4.
Ellie1729 2021-05-19 20:25:47
Hi everyone! I’m Laura, and I’ll be presenting the solution to problem 4. Part (b) will require using some inversive geometry, but we don’t need to know too much except that inversion can turn circles into lines and vice versa. We won’t need that for part (a), though, so let’s start with that.
Ellie1729 2021-05-19 20:25:59
Problem 4 Statement:
Let $S$ be a set of $8$ points in the plane. Call a circle abundant if it passes through at least $4$ points of $S$.
Ellie1729 2021-05-19 20:26:10
Part (a): Show that there can be at most $14$ abundant circles.
Ellie1729 2021-05-19 20:26:22
Problem 4a Solution:
Ellie1729 2021-05-19 20:26:29
For this part, we can just use a counting argument. We know that a circle can be defined by $3$ points. So how many ways are there to choose $3$ points out of the $8$ points in $S$ to form a circle?
RedFireTruck 2021-05-19 20:26:54
$\binom83=56$
kxiang 2021-05-19 20:26:54
8c3
CoolCarsOnTheRun 2021-05-19 20:26:54
8c3=56
peace09 2021-05-19 20:26:54
$\tbinom{8}{3}=56.$
iamhungry 2021-05-19 20:26:54
$\binom{8}{3}=56$
pritiks 2021-05-19 20:26:54
8C3 = 56
Ellie1729 2021-05-19 20:27:01
Right! Since we don’t care about order, there are a total of $\binom{8}{3}=\frac{8\cdot 7 \cdot 6}{3\cdot 2 \cdot 1} = 56$ ways to choose $3$ points from $S$. So there are at most $56$ circles formed by points in $S$.
Ellie1729 2021-05-19 20:27:09
However, if some of the circles are abundant, they would be counted multiple times by this, since abundant circles have more than $3$ points on them. How many times has each abundant circle been counted?
kxiang 2021-05-19 20:27:33
4
RedFireTruck 2021-05-19 20:27:33
$4$ because 4 points
stonkpotato 2021-05-19 20:27:33
4
iamhungry 2021-05-19 20:27:33
$4$ times
peace09 2021-05-19 20:27:33
$\tbinom{4}{3}=4.$
CoolCarsOnTheRun 2021-05-19 20:27:33
4c3=4
artistic_girl 2021-05-19 20:27:33
4
Arcticturn 2021-05-19 20:27:33
4
Ellie1729 2021-05-19 20:27:42
Yes! Since each abundant circle has at least $4$ points on it, there are at least $\binom{4}{3}=4$ ways to choose $3$ points to define an abundant circle. Thus, we’ve counted each abundant circle at least $4$ times.
Ellie1729 2021-05-19 20:27:49
Since we counted $56$ circles total and each abundant circle has been counted at least $4$ times, there can be at most $\frac{56}{4} = 14$ abundant circles.
Ellie1729 2021-05-19 20:28:00
Part (b): Can there be $14$ abundant circles? If not, what is the maximum number possible?
Ellie1729 2021-05-19 20:28:10
To solve this part, we’ll need to know a little about inversion. For the sake of time we won’t go through the proofs here, but let’s review the definition and some key properties of inversion from the handout. You can read through the handout for the proofs.
Ellie1729 2021-05-19 20:28:24
Inversion is sort of like “reflection across a circle.” It takes everything inside the circle and swaps it with everything outside the circle. Points on the circle are fixed, and the center of the circle maps to the “point at infinity.” Inversion can distort images in weird ways! Here are some examples of pictures and their inverted images:
Ellie1729 2021-05-19 20:28:33
http://circleinversions.weebly.com/uploads/1/7/7/6/17765701/3886548.png?698
Ellie1729 2021-05-19 20:28:39
https://encrypted-tbn0.gstatic.com/images?q=tbn:ANd9GcSukX545QA1TgUXrdLpS8zIvycHj2mjS4phcw&usqp=CAU
Ellie1729 2021-05-19 20:28:46
//cdn.artofproblemsolving.com/images/8/c/f/8cf236eca3deed0c388f666d81e63340c08c79e2.jpg
Ellie1729 2021-05-19 20:28:53
More formally, the inverse of a point $P$ about a circle with center $O$ and radius $r$ is the point $P’$ on ray $OP$ such that $OP\cdot OP’ = r^2$.
Ellie1729 2021-05-19 20:29:02
https://encrypted-tbn0.gstatic.com/images?q=tbn:ANd9GcS8t4pefflkgQqUTOSNfp-nWuvBwVAjGOTjjNU_qlc1_pouwccG23ryNb7gqIP3txv-Aso&usqp=CAU
Ellie1729 2021-05-19 20:29:09
Thus, the closer a point is to the circle, the closer its image will be to the circle. Some more examples of points and their inverted images are shown below.
Ellie1729 2021-05-19 20:29:17
https://encrypted-tbn0.gstatic.com/images?q=tbn:ANd9GcRWaLOk9A2SkBZFHQC8U0XpdW2C0qsoxNlpqZ_hjY8TGqjczlojaj-vz355DbYbaHO6O4Q&usqp=CAU
Ellie1729 2021-05-19 20:29:26
https://encrypted-tbn0.gstatic.com/images?q=tbn:ANd9GcQL8Jyv7qgKzu4GTGnxc0cK-T55y0gxbiwHcsk2GonbThMg8tiElmMrnjJZCCVhlyUpdG0&usqp=CAU
Ellie1729 2021-05-19 20:29:37
The key property of inversion we’ll need here is that it can sometimes swap circles with lines. Let’s be a bit more specific about that, though, because there are a few different cases.
Ellie1729 2021-05-19 20:29:48
Inversion sends “generalized circles” to “generalized circles,” where “generalized circles” include lines, because they are like circles through the point at infinity.
Ellie1729 2021-05-19 20:30:01
Let’s say we’re inverting about a circle with center $O$. What should happen to a line through $O$?
CoolCarsOnTheRun 2021-05-19 20:30:30
it doesn't change
bronzetruck2016 2021-05-19 20:30:30
Stays the same
RedFireTruck 2021-05-19 20:30:34
it stays the same?
Ellie1729 2021-05-19 20:30:45
Right! From the definition of inversion, a line through $O$ will be fixed under inversion, so its inverse is the same line through $O$. Every point on the line just goes to another point on the same line.
peace09 2021-05-19 20:30:50
$O$ Disappears?
Ellie1729 2021-05-19 20:31:03
Yes! $O$ gets mapped to the "point at infinity."
Ellie1729 2021-05-19 20:31:19
But the "point at infinity" gets mapped to $O$.
Ellie1729 2021-05-19 20:31:30
https://encrypted-tbn0.gstatic.com/images?q=tbn:ANd9GcRg18fRqAdBYW-7jqINlO7ckwTkhmIu_TSncg&usqp=CAU
Ellie1729 2021-05-19 20:31:40
Okay, what do you think will happen if we invert a circle that doesn’t pass through $O$?
RedFireTruck 2021-05-19 20:32:16
it becomes another circle
CoolCarsOnTheRun 2021-05-19 20:32:16
becomes another circle
artistic_girl 2021-05-19 20:32:16
another circle
SwimWithDolphin 2021-05-19 20:32:16
another circle
Ellie1729 2021-05-19 20:32:26
Yes! We are starting with a circle that doesn’t pass through either $O$ or the point at infinity, so its inverse also won’t pass through either of those points. Thus, the inverse is still a circle not through point $O$.
Ellie1729 2021-05-19 20:32:35
https://lh3.googleusercontent.com/proxy/bsKcErEzXp8raeDDw6ytOM_pfJUXyLnfY_ZrBVdBJheZ3qX--SraFHX2Lh_V89k9qxcyKRZu2fS5QW-RSSgwVPFx
Ellie1729 2021-05-19 20:32:50
Here are some more examples of how this might look. In each case, the solid circles are images of each other under inversion about the dashed circle. If the starting circle is entirely inside the circle of inversion, its inverse will be entirely outside and vice versa. If the starting circle intersects the circle of inversion, so will its inverse.

https://mathworld.wolfram.com/images/eps-gif/InversionCircles_700.gif
Ellie1729 2021-05-19 20:33:36
What do we get if instead we invert a circle through $O$?
CoolCarsOnTheRun 2021-05-19 20:34:09
a line
bronzetruck2016 2021-05-19 20:34:09
Line
SwimWithDolphin 2021-05-19 20:34:09
line?
artistic_girl 2021-05-19 20:34:09
a line
wwwy 2021-05-19 20:34:19
a line
BlueyStudiosYT 2021-05-19 20:34:19
a line
Ellie1729 2021-05-19 20:34:22
Exactly! The inverse of $O$ is the point at infinity, so a circle through $O$ inverts to a “circle” through the point at infinity - that is, a line!
Ellie1729 2021-05-19 20:34:31
Thus, the inverse of a circle through $O$ is a line not through $O$. Likewise, the inverse of a line not through $O$ is a circle through $O$. This is the way we swap lines with circles!
Ellie1729 2021-05-19 20:34:45
http://pi.math.cornell.edu/~dwh/books/eg99/CI/36ede090.png
Ellie1729 2021-05-19 20:34:55
For example, in the picture below, the blue and orange circles invert to lines, because they pass through $O$, while the red circle inverts to a circle, because it doesn’t pass through $O$.
Ellie1729 2021-05-19 20:35:00
https://encrypted-tbn0.gstatic.com/images?q=tbn:ANd9GcS2WKZkD3aEGQ38FReMu5-CazRQ7fzl6D3_hw&usqp=CAU
Ellie1729 2021-05-19 20:35:13
That covers all our cases! Any questions so far?
wwwy 2021-05-19 20:35:49
in the first scenario, what does o disappear mean
Ellie1729 2021-05-19 20:36:08
In all the scenarios, $O$ gets mapped to the point at infinity, which isn't really a point.
dan09 2021-05-19 20:36:27
$O$ goes to infinity
Ellie1729 2021-05-19 20:36:43
Great! Now, onto the solution to part (b).
Ellie1729 2021-05-19 20:36:49
Problem 4b Solution:
Ellie1729 2021-05-19 20:36:56
We claim that in fact the maximum is not $14$ abundant circles, but $12$. To show this, there are two things we need to prove:
Ellie1729 2021-05-19 20:37:06
1. Upper bound: More than $12$ abundant circles is not possible.
Ellie1729 2021-05-19 20:37:16
2. Lower bound: At least $12$ abundant circles is possible.
Ellie1729 2021-05-19 20:37:22
First let’s prove the upper bound. To do so, we’ll think about how many abundant circles can pass through each point in $S$.
Ellie1729 2021-05-19 20:37:29
Let’s count the number of point/circle pairs consisting of a point in $S$ and an abundant circle through that point. If we assume there are $14$ abundant circles and we know each abundant circle passes through $4$ points, how many such pairs would there be?
peace09 2021-05-19 20:38:21
56
dan09 2021-05-19 20:38:21
$14*4=56$?
Ellie1729 2021-05-19 20:38:40
Right, there would be at least $4\cdot 14 = 56$ pairs. So how many circles must there be through each of the $8$ points, on average?
SwimWithDolphin 2021-05-19 20:39:13
7
peace09 2021-05-19 20:39:13
7
dan09 2021-05-19 20:39:13
$\frac{56}{8}=7$
mayasu 2021-05-19 20:39:13
7?
Ellie1729 2021-05-19 20:39:27
Yes! Since there are $8$ points in $S$, on average each point should be on at least $\frac{56}{8} = 7$ abundant circles. In particular, some point would have to be on at least $7$ abundant circles.
Ellie1729 2021-05-19 20:39:40
However, we’ll show that this isn’t possible. Instead, we claim that each point can be on at most $6$ abundant circles. How many point/circle pairs and how many abundant circles would that give us, at most?
RedFireTruck 2021-05-19 20:40:16
$12$
bronzetruck2016 2021-05-19 20:40:16
12
Noah23 2021-05-19 20:40:16
48
artistic_girl 2021-05-19 20:40:16
48
mayasu 2021-05-19 20:40:16
48
peace09 2021-05-19 20:40:16
48
kxiang 2021-05-19 20:40:16
48
CoolCarsOnTheRun 2021-05-19 20:40:21
48 and 12
dan09 2021-05-19 20:40:21
That would be $6*8=48$
Ellie1729 2021-05-19 20:40:55
Exactly! If each of the $8$ points is on at most $6$ circles, there are at most $6 \cdot 8 = 48$ point-circle pairs. Since each abundant circle contains at least $4$ points, there can be at most $\frac{48}{4} = 12$ abundant circles.
dan09 2021-05-19 20:41:10
But we need to prove that each point can be on at most $6$ abundant circles
Ellie1729 2021-05-19 20:41:14
Right!
Ellie1729 2021-05-19 20:41:20
To prove our claim that each point is on at most $6$ abundant circles, let’s imagine inverting about a circle centered at one of the points. After inversion, what will happen to the abundant circles that don’t pass through that point?
kxiang 2021-05-19 20:42:01
still are circles
CircleInvert 2021-05-19 20:42:01
They remain circles
artistic_girl 2021-05-19 20:42:01
another circle
peace09 2021-05-19 20:42:01
They will become... Circles
llddmmtt1 2021-05-19 20:42:01
becomes another circle?
Ellie1729 2021-05-19 20:42:12
Right! These are circles not through the center of inversion, so they’ll stay circles.
Ellie1729 2021-05-19 20:42:20
What about the abundant circles that do pass through our point?
CoolCarsOnTheRun 2021-05-19 20:42:36
become lines
artistic_girl 2021-05-19 20:42:36
lines
bronzetruck2016 2021-05-19 20:42:36
line
dan09 2021-05-19 20:42:36
become a line?
wwwy 2021-05-19 20:42:36
line
kxiang 2021-05-19 20:42:36
turns into lines
llddmmtt1 2021-05-19 20:42:36
become a line
BlueyStudiosYT 2021-05-19 20:42:36
they become a straight line
Batee5a 2021-05-19 20:42:41
Turns to lines!
Ellie1729 2021-05-19 20:42:50
Yes! These circles pass through the center of inversion, so they’ll invert to lines, each containing the images of at least $3$ of the $7$ remaining points.
Ellie1729 2021-05-19 20:42:59
So we can now rephrase our claim as follows:
Ellie1729 2021-05-19 20:43:06
Claim: Through $7$ points, we can draw at most $6$ lines such that each line contains $3$ of the points.
Ellie1729 2021-05-19 20:43:20
Proof: First, we claim that there can be at most $7$ such lines.
Ellie1729 2021-05-19 20:43:31
This uses a similar argument as in part (a). How many lines can there be that pass through at least $2$ of the $7$ points?
peace09 2021-05-19 20:44:15
$\tbinom{7}{2}=21.$
artistic_girl 2021-05-19 20:44:15
21
CoolCarsOnTheRun 2021-05-19 20:44:15
7c2=21
wwwy 2021-05-19 20:44:19
21?
Ellie1729 2021-05-19 20:44:26
Right, there are $\binom{7}{2}=21$ ways to choose $2$ of the $7$ points, and any $2$ points define a line, giving at most $21$ lines passing through at least $2$ of the points.
Ellie1729 2021-05-19 20:44:46
If a line passes through $3$ points, it would be counted $\binom{3}{2} = 3$ times by this, so there can be at most $\frac{21}{3} = 7$ such lines.
Ellie1729 2021-05-19 20:44:57
Note that this actually gives us an alternate proof of part (a)! We’ve shown that each point is on at most $7$ abundant circles, which makes at most $8\cdot 7 = 56$ point-circle pairs, and so at most $\frac{56}{4} = 14$ abundant circles.
Ellie1729 2021-05-19 20:45:14
One way to complete the proof from here is using the Sylvester-Gallai theorem, which says that for any finite set of points in the plane, there must be a line through all of them or a line through exactly $2$ of them.
Ellie1729 2021-05-19 20:45:29
The only way we could have gotten $7$ lines above is if every line containing $2$ of the points actually contains $3$ of them. By the Sylvester-Gallai theorem, that can’t happen, so there can be at most $6$ such lines.
dan09 2021-05-19 20:46:17
*What is the proof of that theorem?
Ellie1729 2021-05-19 20:46:36
We won't have time to go through that here, but you can look it up after class if you're interested!
Ellie1729 2021-05-19 20:46:52
However, if you’re not familiar with that theorem, another way to prove this is by drawing a picture and thinking carefully about how the points and lines could be positioned.
Ellie1729 2021-05-19 20:47:00
If there were $7$ lines, like we saw above, each of them would have to contain exactly $3$ of the points. Likewise, each of the $7$ points would have to lie on exactly $3$ of the lines.
Ellie1729 2021-05-19 20:47:13
Let’s say $A$ is the leftmost point, $B$ is the farthest point from $A$ on the top line, and $C$ is the farthest from $A$ on the bottom line. Then we will get something like the picture below:
Ellie1729 2021-05-19 20:47:18
http://flashman.neocities.org/Courses/7POINT.gif
Ellie1729 2021-05-19 20:47:25
Because of how we defined the points, there must be a third point $E$ in our set on line $AB$ between $A$ and $B$ and a third point $G$ in our set on line $AC$ between $A$ and $C$.
Ellie1729 2021-05-19 20:47:35
There also has to be a point $F$ in our set where line $BC$ intersects the third line through $A$. Finally, lines $BG$, $CE$, and $AF$ must all meet at the final point $D$ in our set.
Ellie1729 2021-05-19 20:47:49
But we can now see that there’s no way for the three points $E, G,$ and $F$ to lie on a line, because $F$ is on the wrong side of $D$ from $E$ and $G$! Thus, there’s no way to add a $7$th line to the picture that passes through $3$ of the points.
Ellie1729 2021-05-19 20:48:02
Another way to prove this is using Ceva’s Theorem and Menelaus's Theorem. Ceva’s Theorem tells us that if the lines $AF$, $BG$, and $CE$ are concurrent, then $$\frac{AG}{GC} \cdot \frac{CF}{FB} \cdot \frac{BE}{EA} = 1.$$
Ellie1729 2021-05-19 20:48:13
But Menalaus’s Theorem tells us that if the points $G, F,$ and $E$ are collinear, then $$\frac{AG}{GC} \cdot \frac{CF}{FB} \cdot \frac{BE}{EA} = -1.$$ We’re using directed lengths here, which is why the product can be negative.
Ellie1729 2021-05-19 20:48:26
The same product can’t equal both $1$ and $-1$, so we’re done!
Ellie1729 2021-05-19 20:48:37
This proves our claim, so we’re done with the upper bound! Any questions so far?
kfcruan 2021-05-19 20:48:55
what is the menalaus's theorm
Ellie1729 2021-05-19 20:49:27
It gives a condition for when three points are collinear, using ratios like above.
kfcruan 2021-05-19 20:49:31
what is the Ceva theorm
Ellie1729 2021-05-19 20:49:50
Ceva's Theorem gives a condition for when three lines are concurrent, using the product of ratios above.
wwwy 2021-05-19 20:49:54
what do you mean by directed lengths
Ellie1729 2021-05-19 20:50:31
Directed lengths basically mean line segments are assigned a direction and can be positive or negative depending on which direction they're in.
mayasu 2021-05-19 20:50:42
what does concurrent mean?
Ellie1729 2021-05-19 20:50:50
Concurrent means the lines pass through the same point.
Batee5a 2021-05-19 20:51:06
Are there any recommended sources to check up on these theorems and practice with them?
Ellie1729 2021-05-19 20:51:31
I think there is probably some information about them on the Aops website, or you can try googling them!
wwwy 2021-05-19 20:51:54
so which direction yields negative sign
Ellie1729 2021-05-19 20:52:07
You get to choose the direction, you just have to be consistent about it.
dan09 2021-05-19 20:52:37
But all we did is prove there isn't 7 lines, not that there is 6 lines
Ellie1729 2021-05-19 20:52:51
That's true! All we've shown so far is that there can be at most $12$ abundant circles.
Ellie1729 2021-05-19 20:53:11
So to finish the problem, we need to show that $12$ abundant circles actually is possible!
Ellie1729 2021-05-19 20:53:36
For the lower bound, we need an example of how we can have $12$ abundant circles. There are a couple possible constructions here.
Ellie1729 2021-05-19 20:53:43
For the construction we’ll show here, we’ll use inversion through a point in $S$ again. This means we’ll start with $7$ points, and either lines through $3$ of the points or circles through $4$ of the points will become abundant circles after inversion.
Ellie1729 2021-05-19 20:53:54
Our construction is to use a triangle, the feet of its altitudes, and its orthocenter as our points.
Ellie1729 2021-05-19 20:53:59
https://ds055uzetaobb.cloudfront.net/brioche/solvable/e2e7d234b2.00b76d6bf0.v1WEbo.png?width=250
Ellie1729 2021-05-19 20:54:05
How many circles are there that pass through at least $4$ of these points?
RedFireTruck 2021-05-19 20:54:50
6 because 6 cyclic quads?
bronzetruck2016 2021-05-19 20:54:50
6
artistic_girl 2021-05-19 20:54:50
6
kxiang 2021-05-19 20:54:50
6?
CircleInvert 2021-05-19 20:54:50
6
Ellie1729 2021-05-19 20:55:02
Right! There are $6$ abundant circles in this picture, specifically $ADHF, CFHE, BEHD, AFEB, BDFC,$ and $CEDA$. We can show each of these is cyclic because they all have two right angles that would be inscribed in the either same arc or opposite arcs.
Ellie1729 2021-05-19 20:55:16
And how many lines are there through at least $3$ of the points?
RedFireTruck 2021-05-19 20:55:54
6
Irving1004 2021-05-19 20:55:54
6
Ellie1729 2021-05-19 20:56:08
Exactly, there are $6$ such lines: the three sides of the triangle and the three altitudes.
Ellie1729 2021-05-19 20:56:15
So when we invert this picture, we’ll get $6 + 6 = 12$ abundant circles. That proves our lower bound!
Ellie1729 2021-05-19 20:56:24
Any questions on Problem 4?
dan09 2021-05-19 20:56:31
Yay, we did it!
mayasu 2021-05-19 20:56:39
i thought it was 8 pts?
Ellie1729 2021-05-19 20:57:08
Right. The 8th point is the one we're inverting about. So in our original picture with the triangle it's actually at the point at infinity.
peace09 2021-05-19 20:57:15
No, but what a nice problem!
Ellie1729 2021-05-19 20:57:19
I think so too
avasireddy056 2021-05-19 20:57:32
about how many lines are through at least 3 of the points
Ellie1729 2021-05-19 20:57:53
I meant on the picture with the triangle, how many lines pass through 3 of the 7 labelled points.
Ellie1729 2021-05-19 20:58:14
Each of those lines also passes through the point at infinity and so will invert to an abundant circle.
Ellie1729 2021-05-19 20:58:50
Alright! On to Problem 5!
MiraBernstein 2021-05-19 20:59:00
Hi everyone, I’m Mira. I’m going to present the solution to Problem #5.
MiraBernstein 2021-05-19 20:59:07
Before I start, I should say that this Math Jam is taking a bit longer than we were expecting. The problems are long and complicated (as you might have noticed when you tried to solve them for the Mathcamp application).
MiraBernstein 2021-05-19 20:59:17
But we definitely expect to be done by about 7 PT/ 10 ET, hopefully earlier. If you need to go before then, you can find the transcript later at https://www.mathcamp.org/qualifying_quiz/solutions/ (or just on the Math Jams archive: https://artofproblemsolving.com/school/mathjams-transcripts).
MiraBernstein 2021-05-19 20:59:26
OK, on to Problem 5!
MiraBernstein 2021-05-19 20:59:42
Problem 5, Part a (paraphrased for brevity):
A small town of 192 people has exactly 2 people with COVID19. We want to find them. Instead of testing everyone individually, we can test up to 32 people at once in a “pooled sample”. If the test comes out negative, none of the people in the pooled sample are sick. If the test comes out positive, one or more of the people are sick, but you don’t know who or how many. Can you find the town’s 2 sick people using 16 tests?
MiraBernstein 2021-05-19 20:59:54
Let’s show that the answer is “yes” by finding an algorithm that works. What’s our first step?
Lilavigne 2021-05-19 21:00:35
separate into groups of 32
artistic_girl 2021-05-19 21:00:35
test 32 people
mayasu 2021-05-19 21:00:48
separate 192 into groups of 32
MiraBernstein 2021-05-19 21:00:51
OK, we’ve divided the 192 people into 6 groups of 32 and tested each group. This used up 6 tests. What are the two possible outcomes?
SHZhang 2021-05-19 21:01:34
One test positive, or 2 tests are positive
Rubikscube3.1415 2021-05-19 21:01:34
1 yes, 2 yeses
bronzetruck2016 2021-05-19 21:01:34
Same group or different group
artistic_girl 2021-05-19 21:01:34
both in the same group or 2 groups
CoolCarsOnTheRun 2021-05-19 21:01:34
only one group is positive or two groups test positive
wwwy 2021-05-19 21:01:34
two positive in one pool or two positive in two pools
kxiang 2021-05-19 21:01:34
two or one groups are positive
Batee5a 2021-05-19 21:01:34
2 separate positives or both being in the same group
twotothetenthis1024 2021-05-19 21:01:34
one group tests positive or two groups
gordonhero 2021-05-19 21:01:34
2 positive groups or 1 positive group
MiraBernstein 2021-05-19 21:01:41
Right, we have two cases:
A) two groups test positive, which means they have one sick person each;
B) one group tests positive, which means it has both of the sick people.
MiraBernstein 2021-05-19 21:01:49
Let’s deal with case A first. Time for a lemma!
MiraBernstein 2021-05-19 21:01:56
Lemma 1: If you know that a group of $2^n$ has exactly one sick person, you can find that person with $n$ tests.
MiraBernstein 2021-05-19 21:02:05
Proof: Split your $2^n$ people in half and test the first half. The result of the test tells you whether the sick person is in that half or in the other half.
MiraBernstein 2021-05-19 21:02:13
In either case, you’ve used 1 test to narrow down your list of ``suspects” from $2^n$ to $2^{n-1}$. Keep going this way, and you’ll use $n$ tests to narrow it down to 1 person -- the sick one.                 QED
wwwy 2021-05-19 21:02:22
I use binary
MiraBernstein 2021-05-19 21:02:28
Exactly!
MiraBernstein 2021-05-19 21:02:34
(For those who know some CS: this is just binary search. For those who know induction: this is basically a proof by induction, but it’s such a simple case that I skipped the formalism. The next proof will be more formal.)
MiraBernstein 2021-05-19 21:02:45
Getting back to our situation: If two of your 6 groups of 32 test positive, you’ll need 5 more tests to find the sick person in each group. So how many tests have you used in total?
bronzetruck2016 2021-05-19 21:03:15
16!!!
Rubikscube3.1415 2021-05-19 21:03:15
16, so it works
gordonhero 2021-05-19 21:03:15
16 tests
twotothetenthis1024 2021-05-19 21:03:15
16
MiraBernstein 2021-05-19 21:03:22
Yup, $32 = 2^5$, so we’ll need 5 tests for each group that tested positive. So we can find the two sick people using $6+5+5 = 16$ tests, as required.
MiraBernstein 2021-05-19 21:03:34
Now let’s do case B: both sick people are in the same group of 32.



Time for another lemma! This one is a little more complicated, so I’ll use formal induction:
MiraBernstein 2021-05-19 21:03:45
Lemma 2: If you know that a group of $2^n$ people has 2 sick people, you can always find them using $2n$ tests (or fewer).
MiraBernstein 2021-05-19 21:03:56
Proof: Induction on $n$.

Base case: When $n=1$, you have 2 people and you already know that both are sick, so you need 0 tests. And indeed, $0<2\cdot 1$.
MiraBernstein 2021-05-19 21:04:07
Inductive hypothesis: Suppose the statement is true for $n = k-1$. Now say you have a group of $2^k$ people in which you know that 2 are sick. We need to show that you can find those 2 people with $2k$ tests.
MiraBernstein 2021-05-19 21:04:18
Split your $2^k$ people in half (two groups of $2^{k-1}$) and test both halves. We’ve now used 2 tests. Like before, there are two possibilities:
MiraBernstein 2021-05-19 21:04:28
Case 1: Both halves test positive. [\i] This means each of them contains one sick person. What can we say now?
MiraBernstein 2021-05-19 21:04:44
(oops, sorry about the formatting)
MiraBernstein 2021-05-19 21:05:03
But the question still stands
the_mathmagician 2021-05-19 21:05:18
We can use our first lemma
dan09 2021-05-19 21:05:18
Do another binary search on each one
twotothetenthis1024 2021-05-19 21:05:18
this becomes essentially case A once one test has been done per group (so it works!)
peace09 2021-05-19 21:05:18
It takes $k$ tests to identify a sick person in each group, and since we have two groups, we need $2k$ tests.
MiraBernstein 2021-05-19 21:05:27
By Lemma 1, we’ll need $k-1$ more tests for each of the two groups. So the total number of tests we used for our group of $2^k$ is:
MiraBernstein 2021-05-19 21:05:41
$\quad$* 2 at the beginning, to test each half.

$\quad$* $k-1$ to find the sick person in the first half (by Lemma 1)

$\quad$* $k-1$ to find the sick person in the second half (by Lemma 1)
MiraBernstein 2021-05-19 21:05:48
This adds up to $2k$ tests, as required.
MiraBernstein 2021-05-19 21:06:00
Case 2: Only one of the two groups of $2^{k-1}$ tests positive. Then this group has both sick people in it. What do we do now?
MiraBernstein 2021-05-19 21:06:14
(hey look, better formatting!)
twotothetenthis1024 2021-05-19 21:06:48
then split it in half and try again!
dan09 2021-05-19 21:06:48
do another binary test only on the group with both!
Batee5a 2021-05-19 21:06:48
Split in half again and repeat the strategy
gxxk 2021-05-19 21:06:48
Keep on binary searching
the_mathmagician 2021-05-19 21:06:48
Repeat, split in half, check again, recursively, until you either chop it into 2 people, or you get 2 people in different groups sick, then proceed with the first lemma.
MiraBernstein 2021-05-19 21:06:57
You guys are all on the right track
MiraBernstein 2021-05-19 21:07:05
But remember, we're in the middle of a proof by induction!
peace09 2021-05-19 21:07:32
Apply the inductive hypothesis!
MiraBernstein 2021-05-19 21:07:41
Hurrah, apply the inductive hypothesis!



We have a group of $2^{k-1}$ people, so we know we can find the 2 sick people in $2(k-1)$ tests.
MiraBernstein 2021-05-19 21:07:53
(That IS the inductive hypothesis.)
MiraBernstein 2021-05-19 21:08:04
And we already used 2 tests.
the_mathmagician 2021-05-19 21:08:20
$2k-2+2=2k$
MiraBernstein 2021-05-19 21:08:27
yup!
MiraBernstein 2021-05-19 21:08:33
Applying this to Problem 5a, Case B: If one of your 6 groups of 32 tests positive, Lemma 2 says you can find the two sick people in that group in 10 tests. So in total, you will have used 6+10 = 16 tests, as required.
MiraBernstein 2021-05-19 21:08:47
Questions on 5a?
MiraBernstein 2021-05-19 21:09:05
OK, onto 5b.
MiraBernstein 2021-05-19 21:09:17
Part b (paraphrased): Now you need to find 2 people out of 1000, but you have to specify your pooled samples ahead of time. You can’t decide whom to test next based on the results of earlier tests.

Find the minimum number of pooled samples needed to identify who is sick, to within a factor of 2. That is, find an $n$-sample strategy, and prove that $n/2$ samples are not enough, for some value of $n$.
MiraBernstein 2021-05-19 21:09:45
This is a very different problem!
MiraBernstein 2021-05-19 21:09:52
Finding the actual minimal number of tests needed in this case is an open problem! We ourselves don’t know the answer.
MiraBernstein 2021-05-19 21:10:01
When we put this problem on the Quiz, here’s what we knew how to do:

$\quad$* We could prove that you need at least 61 tests (a lower bound) .
$\quad$* We knew a strategy for finding the 2 sick people using 93 tests (an upper bound) .
MiraBernstein 2021-05-19 21:10:15
These two things are enough to answer the specific question that we asked on the Quiz, since $n=93$ works and $n/2=47 < 61$ doesn’t work. That’s why we posed the problem this way!
MiraBernstein 2021-05-19 21:10:26
So we knew that the true minimum was somewhere between 61 and 93, but we didn’t know how to narrow it down within that interval.
MiraBernstein 2021-05-19 21:10:35
We love putting open problems on the Qualifying Quiz, because we’re always hoping that someone will come up with a better partial solution than ours.



Almost always, somebody does -- including this time! You guys (collectively) beat us by a teeny-tiny bit.
MiraBernstein 2021-05-19 21:10:47
No one came up with a strategy with fewer than 93 tests, but an applicant named Andrew, from Boyds, MD proved that you need at least 62 tests.



Andrew used essentially the same approach as us (and as the rest of the people who solved this problem), but he managed to raise the lower bound from 61 to 62 with some clever casework.
MiraBernstein 2021-05-19 21:10:58
I won’t present Andrew’s proof here, because the casework is a little too messy to do in real time. But I did want to give him a shoutout!
MiraBernstein 2021-05-19 21:11:07
OK, to recap: the solution to Problem 5b has two parts -- proving a lower bound and constructing an upper bound. Let’s do the lower bound first:
MiraBernstein 2021-05-19 21:11:20
Claim: You need at least 61 tests to find the two sick people.
MiraBernstein 2021-05-19 21:11:33
Proof: Suppose you can do it with $k$ tests. We want to show that $k \geq 61$.

Here’s the key observation: You can’t have two people in exactly the same set of tests. Why not?
wbender25 2021-05-19 21:12:47
We wouldn't be able to tell which one is sick.
dolphin06670 2021-05-19 21:12:47
You wouldn't know which one is sick?
SHZhang 2021-05-19 21:12:50
Let these people be A and B, choose a third person C, then A and C infected is indistinguishable from B and C infected
Batee5a 2021-05-19 21:13:05
if one is infected and the other isn't there'll be no way to differentiate them
MiraBernstein 2021-05-19 21:13:28
Right! If exactly one of those two people is sick, you have no way of knowing from the test results which one it is. I’m going to call this the distinct test principle .
MiraBernstein 2021-05-19 21:13:40
OK, so what’s the maximal number of people that can be in zero tests?
wwwy 2021-05-19 21:14:21
you mean zero-positive test?
MiraBernstein 2021-05-19 21:14:30
No, I mean people who are not tested at all
CircleInvert 2021-05-19 21:15:06
1
artistic_girl 2021-05-19 21:15:06
1
SHZhang 2021-05-19 21:15:06
one person
conantwiz2023 2021-05-19 21:15:06
1&
MiraBernstein 2021-05-19 21:15:11
Yup, there’s only one set of size 0 (the empty set). So by the distinct test principle, we can leave at most 1 person untested. At least 999 people must be tested.
MiraBernstein 2021-05-19 21:15:23
A lot of people said 0, but it IS OK to leave one person out
MiraBernstein 2021-05-19 21:15:41
if you find only one sick person among the other 999, then you know the one you left out is also sick
MiraBernstein 2021-05-19 21:16:07
What’s the maximal number of people whom we can test just once?
MiraBernstein 2021-05-19 21:17:15
Remember, we are assuming we're doing $k$ tests total.
MiraBernstein 2021-05-19 21:17:26
(We're trying to show $k>=61$)
MiraBernstein 2021-05-19 21:18:01
What happens if we put two people in the same test, and that's the only test for both of them?
ChrisalonaLiverspur 2021-05-19 21:18:45
Well if its positive, how can we find it out for them?
peace09 2021-05-19 21:18:45
If positive then how do we know if 1 or 2 of those people are positive?
MiraBernstein 2021-05-19 21:18:59
Right, so that doesn't work: we can't have two once-tested people in the same test.
MiraBernstein 2021-05-19 21:19:10
So: What’s the maximal number of people whom we can test just once?
MiraBernstein 2021-05-19 21:20:06
A lot of people are saying 1, but that's 1 per each test. Remember, we're doing $k$ tests!
dolphin06670 2021-05-19 21:20:34
k?
Eng123 2021-05-19 21:20:34
k.
wwwy 2021-05-19 21:20:34
k
artistic_girl 2021-05-19 21:20:34
k
MiraBernstein 2021-05-19 21:20:40
Yes! Since there are $k$ tests, the distinct test principle tells us that there can be at most $k$ people who are tested once (and all of them have to be in different tests). If there were two such people in the same test, then there would be no way for us to tell them apart.
MiraBernstein 2021-05-19 21:21:10
OK, so at most 1 person gets tested zero times and at most $k$ people get tested once.
MiraBernstein 2021-05-19 21:21:17
OK, say there are $a$ people who get tested only once. (We just showed that $a \leq k$.) Then we’re testing at least $999-a$ people two or more times.
MiraBernstein 2021-05-19 21:21:25
That’s a total of at least $a + 2(999 - a) = 1998 - a$ individual samples to put into our tests.
MiraBernstein 2021-05-19 21:21:38
Since $a \leq k$, we conclude that the number of individual samples we’ll need to process in our $k$ pooled tests is at least $1999 - k$.
MiraBernstein 2021-05-19 21:21:50
Now, how can we get an inequality in the other direction?
MiraBernstein 2021-05-19 21:22:39
We can include at most 32 individual samples in one test
MiraBernstein 2021-05-19 21:22:54
So how many individual samples can we process total?
MiraBernstein 2021-05-19 21:23:55
Uh-oh, I think I'm losing you guys.
MiraBernstein 2021-05-19 21:24:03
Forget the $1999-k$.
MiraBernstein 2021-05-19 21:24:09
We're doing $k$ tests.
MiraBernstein 2021-05-19 21:24:19
Each one can process at most 32 individual samples
MiraBernstein 2021-05-19 21:24:33
So what's an upper bound on the number of individual samples?
Batee5a 2021-05-19 21:24:47
why 32?
MiraBernstein 2021-05-19 21:24:59
That's in the problem statement
MiraBernstein 2021-05-19 21:25:04
(sorry, it was in Part a)
MiraBernstein 2021-05-19 21:25:44
In our $k$ tests, we can't include more than $32k$ individual samples
artistic_girl 2021-05-19 21:26:02
$32 \dot k$
MiraBernstein 2021-05-19 21:26:06
Yup!
MiraBernstein 2021-05-19 21:26:32
But we said that we had to test at least $1998-k$ individual samples.
MiraBernstein 2021-05-19 21:26:47
(Sorry, the 1999 before was a typo.)
MiraBernstein 2021-05-19 21:27:14
This gives us the inequality $32k \geq 1998 - k$.
artistic_girl 2021-05-19 21:27:21
1998 / 33
MiraBernstein 2021-05-19 21:27:25
Right!
MiraBernstein 2021-05-19 21:27:32
Solving for $k$, we get $k \geq 1998/33 \approx 60.5$.

Since $k$ is an integer, we conclude that $k \geq 61$, QED.
MiraBernstein 2021-05-19 21:27:45
How're you guys doing? Any questions?
peace09 2021-05-19 21:28:07
When you say it, it makes sense, but I can't find it on my own! :o
Eng123 2021-05-19 21:28:07
Confused but alive. Doable.
MiraBernstein 2021-05-19 21:28:10
MiraBernstein 2021-05-19 21:28:38
OK, that was the lower bound: we know we need at least 61 tests.
MiraBernstein 2021-05-19 21:29:04
(People who are confused: the ipper bound proof is easier to follow.)
MiraBernstein 2021-05-19 21:29:09
*upper
MiraBernstein 2021-05-19 21:29:20
So let’s show how we can actually find the 2 sick people using $93$ tests.
SHZhang 2021-05-19 21:29:39
Put the people into a 32 by 32 grid
MiraBernstein 2021-05-19 21:30:12
One approach is to imagine our 1000 people arranged in a $32 \times 32$ square. (Some of the places in the square will be unoccupied, since $32 \cdot 32 > 1000$. It doesn’t matter.)
MiraBernstein 2021-05-19 21:30:19
We can now refer to individuals by their location in this square array: for example $(a,b)$ is the individual whose sample is in row $a$ and column $b$.
MiraBernstein 2021-05-19 21:30:26
Now we can do a pooled test on the people in each column and a pooled test on the people in each row, for a total of $32 + 32 = 64$ tests. How many positive row-tests will we get?
SHZhang 2021-05-19 21:31:09
Either 1 or 2
gordonhero 2021-05-19 21:31:09
2 or 1
MiraBernstein 2021-05-19 21:31:19
Right, it can be 1 or 2, depending on whether the sick people are in the same row or in different rows. And similarly, we’ll get either 1 or 2 positive column tests.
MiraBernstein 2021-05-19 21:31:27
So there are three possibilities:

Outcome 1: Row $a$ tests positive, columns $c$ and $d$ test positive. What does that tell us?
MiraBernstein 2021-05-19 21:31:46
(What are the coordinates of the sick people?)
Batee5a 2021-05-19 21:32:03
(a,c) and (a,d) are infected
peace09 2021-05-19 21:32:03
$(a,c)$ and $(a,d)$
gordonhero 2021-05-19 21:32:03
there are two people (a,c) and (a,d) who are sick
wbender25 2021-05-19 21:32:03
$(a,c)$ and $(a,d)$ are sick.
SHZhang 2021-05-19 21:32:03
(a, c), (a, d)
Mathdreams 2021-05-19 21:32:11
(a,c) and (a,d)
MiraBernstein 2021-05-19 21:32:19
Right, the sick people have to be $(a,c)$ and $(a,d)$.

Outcome 2: Rows $a$ and $b$ test positive, column $c$ tests positive. This tells us that the sick people must be $(a,c)$ and $(b,c)$.
MiraBernstein 2021-05-19 21:32:34
Now here’s the interesting case:

Outcome 3: Rows $a$ and $b$ test positive, columns $c$ and $d$ test positive. What does that tell us?
w_aops 2021-05-19 21:33:16
Either (a, d) and (b, c); or (a, c) and (b, d)
wbender25 2021-05-19 21:33:16
Either $(a,c)$ and $(b,d)$ are sick, or $(a,d)$ and $(b,c)$ are sick.
SHZhang 2021-05-19 21:33:16
Either (a, c) and (b, d), or (a, d) and (b, c)
Batee5a 2021-05-19 21:33:31
(a,c), (b,d) or (a,d), (b,c)
MiraBernstein 2021-05-19 21:33:34
Right, there are two options for who the sick people are. It could be either the pair $(a,c)$ and $(b,d)$ or the pair $(a,d)$ and $(b,c)$.

Here’s a picture ($10\times10$ instead of $32\times32$, but same idea):

https://www.mathcamp.org/files/yearly/2021/mathjams/QQ21_5b_1.jpg
MiraBernstein 2021-05-19 21:33:51
We know that the sick people are either the two blue squares or the two green squares. Now we just need some way to tell which pair of squares it is!

But remember, we’re designing all these tests in advance, so we don’t know ahead of time which two pairs we’ll be confused between. What can we do?
BobsonJoe 2021-05-19 21:34:39
test the diagonals
Batee5a 2021-05-19 21:34:39
diagonals?
gordonhero 2021-05-19 21:34:39
test diagonals?
MiraBernstein 2021-05-19 21:34:52
Yes! I think this is the hardest idea to come up with.
MiraBernstein 2021-05-19 21:35:02
So, maybe like this:

https://www.mathcamp.org/files/yearly/2021/mathjams/QQ21_5b_2.jpg

We do a pooled test on the squares through which each diagonal passes. (I drew all my diagonals going from top-left to bottom-right, but you can also have them going in the other direction.)
MiraBernstein 2021-05-19 21:35:10
In our picture, if the people in the bright-blue squares are sick, then the two light-blue diagonals will test positive. If the people in the bright-green squares are sick, then the light-green diagonal will test positive.
Annabeth_ 2021-05-19 21:35:17
why diagonals?
MiraBernstein 2021-05-19 21:35:23
Just because it works!
MiraBernstein 2021-05-19 21:35:38
Notice that the two sick people might be on the same diagonal as each other (like the green squares) or they might not (like the blue squares). It doesn’t matter.

The important thing is that the blue squares can never be on the same diagonal(s) as the green squares! So knowing which diagonal(s) tested positive will tell you which of the two possible pairs of people (green or blue) is sick.
peace09 2021-05-19 21:35:45
Why do we do it in the first place though? What's the motivation?
MiraBernstein 2021-05-19 21:36:06
Well, we have to distinguish between the two pairs of squares.
MiraBernstein 2021-05-19 21:36:24
Rows and columns don't work any more, but this does.
MiraBernstein 2021-05-19 21:36:44
You could do other things -- e.g. diagonals of other slopes could also work
MiraBernstein 2021-05-19 21:37:00
Anything where the blue and green squares would not be in the same tests
BlueyStudiosYT 2021-05-19 21:37:04
Who came up with the idea to use diagnals?
MiraBernstein 2021-05-19 21:37:23
Apparently many people independently!
MiraBernstein 2021-05-19 21:37:35
But it's a very clever idea that probably took them a while to come up with.
MiraBernstein 2021-05-19 21:37:40
It certainly took me a while!
MiraBernstein 2021-05-19 21:37:50
OK, so we’ve decided to test all the diagonals. How many tests does this take? How many diagonals are there in our picture (the $10\times10$ version)?
peace09 2021-05-19 21:38:16
19.
gordonhero 2021-05-19 21:38:16
10 + 9 = 19
MiraBernstein 2021-05-19 21:38:27
Right, the $10\times10$ square has $10+10-1$ diagonals. And in the same way, the $32\times32$ square has 63 diagonals.



But we already did 64 tests on the rows and columns, so it looks like our strategy requires 127 tests. That’s too many! I promised you 93.
MiraBernstein 2021-05-19 21:38:43
Let’s get it down to 96 tests first, and then we can talk about how to get further down to 93. What can we do to decrease the number of diagonal tests from 63 to 32?
Smileyklaws 2021-05-19 21:38:54
diagonals can wrap around
MiraBernstein 2021-05-19 21:39:08
Yes! Here’s a picture:

https://www.mathcamp.org/files/yearly/2021/mathjams/QQ21_5b_3.jpg
MiraBernstein 2021-05-19 21:39:18
We let the diagonals “wrap around” and combine the pieces of the same color into the same pooled test. So all the squares covered by the red diagonal are in one test, all the squares covered by the orange diagonal are in another test, etc.
MiraBernstein 2021-05-19 21:39:30
Another clever idea!
MiraBernstein 2021-05-19 21:39:36
You can think of this as combining all squares $(x,y)$ that satisfy the linear equation

$$

x+y = i mod 32,

$$

for each value $i = 0,1,\dots,31$. (Or, in our picture, mod 10.)



For example, if the bottom left square is $(0,0)$, then what is $i$ for the orange diagonal?
MiraBernstein 2021-05-19 21:40:17
$$

x+y = i \mod 32,

$$
MiraBernstein 2021-05-19 21:40:29
https://www.mathcamp.org/files/yearly/2021/mathjams/QQ21_5b_3.jpg
peace09 2021-05-19 21:41:28
6
MiraBernstein 2021-05-19 21:41:30
If the bottom left is $(0,0)$, then the orange diagonal has points $(0,6)$, $(1,5)$, etc.
MiraBernstein 2021-05-19 21:41:49
The orange diagonal consists of all squares $(x,y)$ satisfying $x+y = 6 \mod 10$.
MiraBernstein 2021-05-19 21:41:58
If you let the diagonals wrap around this way, a $32\times 32$ square will have only $32$ of them, since $i$ ranges from 0 to 31.



So now we’ve used 32 row tests, 32 column tests, and 32 diagonal tests, for a total of 96 tests.
MiraBernstein 2021-05-19 21:42:07
This already solves the problem as posed on the Quiz, since 96 works and $96/2 = 48$ doesn’t work. But just for fun, how can we get it further down to 93?
MiraBernstein 2021-05-19 21:42:39
We're running late, so I'll just give it away.
MiraBernstein 2021-05-19 21:42:47
We can skip one row, one column, and one diagonal.



Then, for example, if only one row tests positive, you don’t know if it’s because both sick people are in that row or because one is in that row and the other is in the row that wasn’t tested. But the columns and the diagonals will always let you disambiguate that.



There are a bunch of cases to check here, but it always works.
MiraBernstein 2021-05-19 21:43:04
(Check it for homework! )
MiraBernstein 2021-05-19 21:43:13
Any questions about this strategy?
peace09 2021-05-19 21:43:39
Nope! (Except the 93 part... I'll have to take a look at that some other time )
MiraBernstein 2021-05-19 21:43:54
It's kind of like the way we could leave one person out of 1000 untested.
MiraBernstein 2021-05-19 21:44:08
We can leave one row, one column, and one diagonal untested.
SHZhang 2021-05-19 21:44:14
The wraparound idea is beautiful! I didn't find it, but managed to get down to 119 tests by moving people away from the corner...
MiraBernstein 2021-05-19 21:44:22
yes, 119 was good enoughg!
MiraBernstein 2021-05-19 21:44:30
Before we wrap up with Problem 5b, I wanted to mention a really nice alternative testing strategy that a lot of people used:



Choose three pairwise relatively prime numbers (e.g. 32, 33, and 35) and do three sets of pooled tests: one set pooling all numbers that are the same mod 32, one for mod 33, and one for mod 35. Then apply the Chinese Remainder Theorem. Do a little modular arithmetic and you’re done.
MiraBernstein 2021-05-19 21:44:41
If you think about it, this is actually very similar to our row-column-diagonal strategy in spirit, even though one uses geometry and one uses number theory.
MiraBernstein 2021-05-19 21:45:18
OK, on to 5c! (Sorry, this is running so late folks!)
MiraBernstein 2021-05-19 21:45:36
(But 5c is very cool, so I hope you'll stay!)
MiraBernstein 2021-05-19 21:45:42
Problem 5c (paraphrased). A city wants to test 10,000 people. You know that each person has a 5% chance of being sick, but you don’t know the exact number of sick people. This time, you don’t have to design the tests in advance (as in Part b); you can base each test on your previous results (as in Part a).

Find a strategy to reduce the number of tests that need to be done; it doesn't need to be provably optimal, but do your best. On average, how many tests will be required?
MiraBernstein 2021-05-19 21:46:01
Problem 5c was probably the most fun problem on the Quiz for us to read, and also the hardest for us to evaluate!
MiraBernstein 2021-05-19 21:46:09
People came up with so many different strategies and really clever ways of computing the expected number of tests for their strategies. But we also had to scrutinize each strategy carefully, because some of them had subtle probability mistakes that were really hard to spot.
MiraBernstein 2021-05-19 21:46:21
And once again, you guys collectively beat us! Three applicants independently came up with a really elegant strategy that was both simpler and more efficient than our own best strategy.
MiraBernstein 2021-05-19 21:46:29
This is the strategy I’ll present today -- the most efficient strategy of all the ones that were submitted by our 540 applicants and ourselves!
MiraBernstein 2021-05-19 21:46:37
It is due to:

$\quad$Sumedh, from San Jose, CA

$\quad$Andrew, from Pennington, NJ

$\quad$Giorgi, from Tbilisi, Republic of Georgia
MiraBernstein 2021-05-19 21:46:45
(There were also many other awesome approaches that were only a tiny bit less efficient. It makes me want to teach a whole Mathcamp class on the different solutions to this problem -- both the correct ones and the very subtly wrong ones!)
GeronimoStilton 2021-05-19 21:47:02
wow, 540 applicants!
MiraBernstein 2021-05-19 21:47:07
Indeed.
MiraBernstein 2021-05-19 21:47:09
Ready for the “winning” strategy? Here we go.
MiraBernstein 2021-05-19 21:47:19
First, divide your 10000 people into 625 groups of 16. We’ll call these 625 groups the Big Groups..

(BTW, by “groups” I just mean “sets”; this has nothing to do with group theory. )
w_aops 2021-05-19 21:47:23
tests of 32 people are not necessarily optimal?
MiraBernstein 2021-05-19 21:47:31
Yeah, we'll see why soon.
MiraBernstein 2021-05-19 21:47:41
For each Big Group, do the following:
MiraBernstein 2021-05-19 21:47:53
$\quad$* Let $G_0$ be the entire group.

$\quad$* Do one test on all of $G_0$:

$\qquad$* If it’s negative, you’re done with this Big Group. Go on to the next one.

$\qquad$* If it’s positive, use the approach from Lemma 1 in 5a (binary search) to find just one sick person in $G_0$.
MiraBernstein 2021-05-19 21:48:01
What’s the fastest way to do this?
MiraBernstein 2021-05-19 21:48:10
(to find one sick person)
gordonhero 2021-05-19 21:48:48
binary search
peace09 2021-05-19 21:48:48
$\left \lceil \log_{2} G_0 \right \rceil.$
MiraBernstein 2021-05-19 21:49:00
Right, it’s just like Lemma 1 in 5a: you split your group in half and test the first half. If it tests positive, proceed with that half. If it tests negative, proceed with the other half. Keep splitting in half until you’re done.



So you’ll test 8 people, then 4 people, then 2 people, then 1 person: 4 tests.
MiraBernstein 2021-05-19 21:49:20
( $|G_0| = 4$)
MiraBernstein 2021-05-19 21:49:27
Are we done with this Big Group now? Can we move to the next one?
artistic_girl 2021-05-19 21:50:07
how about more than 1 sick
SHZhang 2021-05-19 21:50:07
No - need to find all infected people in group
MiraBernstein 2021-05-19 21:50:13
That’s right, there may be other sick people in our Big Group $G_0$. So let’s remove the sick person we just, and call the remaining group of 15 people $G_1$.
MiraBernstein 2021-05-19 21:50:23
Now we go back and repeat all the preceding steps with $G_1$:

$\quad$* Do one test on all of $G_1$:
$\qquad$* If it’s negative, you’re done with this Big Group. Go on to the next one.
$\qquad$* If it’s positive, find one sick person in $G_1$.
MiraBernstein 2021-05-19 21:50:36
How many tests does it take to find one sick person in $G_1$?
GeronimoStilton 2021-05-19 21:50:55
4?
twotothetenthis1024 2021-05-19 21:50:55
4
MiraBernstein 2021-05-19 21:51:00
Sort of.
MiraBernstein 2021-05-19 21:51:02
$G_1$ has 15 people, which is not a power of 2. But you can still keep splitting it in (approximately) half, over and over until you reach 1.
MiraBernstein 2021-05-19 21:51:11
Depending on luck, it might take you either 3 or 4 splits (tests) until you find the sick person. But it’s definitely no worse than with 16 people, so we know it’s at most 4 tests.
MiraBernstein 2021-05-19 21:51:22
OK, say we’ve found a sick person in $G_1$. What do you think we should do next?
gordonhero 2021-05-19 21:51:46
check g_2 which as 14 people
twotothetenthis1024 2021-05-19 21:51:46
take them out, then we have G_3 and repeat the process
MiraBernstein 2021-05-19 21:51:50
You guessed it! We form $G_2$ by removing the sick person we just found from $G_1$. And then we just keep going in the same way:
MiraBernstein 2021-05-19 21:52:00
$\quad$* Do a pooled test on all of $G_i$, for whichever $i$ you’re on.
$\qquad$* If it’s negative, you’re done with this Big Group. Go on to the next one.
$\qquad$* If it’s positive:
$\quad\qquad$* Find one sick person in $G_i$.
$\quad\qquad$* Form $G_{i+1}$ by removing this sick person from $G_i$.
$\quad\qquad$* Do it all again for $G_{i+1}$.
MiraBernstein 2021-05-19 21:52:44
So we start with a group of 16. We keep finding sick people one at a time, removing them, and checking whether the remaining group still tests positive.
MiraBernstein 2021-05-19 21:53:00
When does this process end?
Batee5a 2021-05-19 21:53:28
when the whole $G_i$ tests negative
MiraBernstein 2021-05-19 21:53:34
Once some $G_i$ tests negative, we can be sure that we’ve found and removed all the sick people from our original big group $G_0$. So we move on to the next Big Group.
MiraBernstein 2021-05-19 21:53:44
We go through all the Big Groups this way and we’re done. That’s the whole strategy.
llddmmtt1 2021-05-19 21:54:07
what about the 5%
MiraBernstein 2021-05-19 21:54:21
Right, the 5% becomes relevant when you try to compute the expected number of tests for this strategy
MiraBernstein 2021-05-19 21:54:33
I don’t know about you, but when I first read this strategy, I thought: “This is SO inefficient!”
MiraBernstein 2021-05-19 21:54:43
It seems so silly to retest all of $G_{i+1}$ each time!
MiraBernstein 2021-05-19 21:54:58
For instance, why are we retesting all 15 people in $G_1$ as if we know nothing about them? Most likely, we’ve already ruled some of them out when we were looking for our first sick person in $G_0$.



E.g., if the first group of 8 that we tested came out negative, why include these 8 people in $G_1$?
MiraBernstein 2021-05-19 21:55:10
So it’s clear that this strategy can be improved, possibly by a lot.

The problem is, adding all that extra complexity makes it more difficult to compute the expected number of tests that will be required. This is why all three people who came up with this strategy chose to keep it as simple as possible.
MiraBernstein 2021-05-19 21:55:20
But it turns out that even this simplistic version, which seems so inefficient at first glance, does better in expectation than any other strategy proposed by our 540 applicants (or ourselves)!
MiraBernstein 2021-05-19 21:55:51
I think in the interest of time, I should stop here?
MiraBernstein 2021-05-19 21:56:12
Computing the expected number is the best part, but we really need to get to problem 6
MiraBernstein 2021-05-19 21:56:35
I'll stay afterwards for anyone who wants to see it, but I feel bad about Misha waiting to do #6.
MiraBernstein 2021-05-19 21:56:39
So let's pass it on to him!
MiraBernstein 2021-05-19 21:56:54
The answer is 3125, as an upper bound
Misha 2021-05-19 21:57:03
Okay. Hi everyone! I'm Misha, and I will tell you how to solve problem #6.
Misha 2021-05-19 21:57:09
Problem 6. Infinitely many campers line up in a row for a Competitive Hanabi Set (CHS) tournament. Each camper has an unknown, numerical skill level in playing CHS. A more skilled player will always beat a less skilled one, but the game is very subtle; it is impossible to compare skill levels, except in a one-on-one match. We will write $A \prec B$ if camper $A$ loses to camper $B$ in CHS (which means that $A$'s skill level is a lower number than $B$'s).
Misha 2021-05-19 21:57:15
A subsequence of $n$ campers is an $n$-tuple of campers $(A_1, A_2, \dots, A_n)$ such that the campers are standing in the infinite row in that order, not necessarily consecutively. ($A_1$ is to the left of $A_2$, who is to the left of $A_3$, and so on.)
Misha 2021-05-19 21:57:23
(a) You must schedule CHS games between the campers to find a subsequence $(A_1,A_2,A_3)$ of three campers such that either $A_1 \prec A_2 \prec A_3$ or $A_1 \succ A_2 \succ A_3$. You can use the results of each game to schedule the next.
Misha 2021-05-19 21:57:31
How can you minimize the number of games required in the worst case? Prove that your answer is optimal.
Misha 2021-05-19 21:58:14
(I'll give you a moment to refresh your memory of this problem. Another optimization problem - like problem #3 and problem #5 were.)
Misha 2021-05-19 21:58:37
Problem 6(a) solution.
Misha 2021-05-19 21:58:46
Let's first go over the definitions and how to think about them. Here's a diagram of the outcomes of a few games: an arrow $A \to B$ means $A$ beats $B$, and I colored arrows red if they point right and blue if they point left.
Misha 2021-05-19 21:58:51
//cdn.artofproblemsolving.com/images/0/7/c/07ce606b500c2a83c2d51fc7d51ca01b7f329f23.png
Misha 2021-05-19 21:58:58
Just to check that everyone's on the same page, where are the subsequences we want?
artistic_girl 2021-05-19 22:00:07
ACD or BCE
BobsonJoe 2021-05-19 22:00:07
ACD, ECB
llddmmtt1 2021-05-19 22:00:07
ACD and BCE?
Misha 2021-05-19 22:00:15
We are happy with $(A,C,D)$ because $A \succ C \succ D$, and with $(B,C,E)$ because $B \prec C \prec E$. We are not happy with $(A,C,B)$ or $(E,C,D)$, because they're not subsequences!
Misha 2021-05-19 22:00:29
Anyway, we can find one of these subsequences in 4 games. Here's one way to do that. Begin with two games $A$ vs. $B$ and $D$ vs. $E$, which can have four outcomes:
Misha 2021-05-19 22:00:35
//cdn.artofproblemsolving.com/images/6/9/7/6971b6b6f6948321a83691ee7841f70d71e44286.png
Misha 2021-05-19 22:00:50
What should we do in case (ii) - or case (iii), which is symmetric? Here, one more game is enough.
BobsonJoe 2021-05-19 22:01:58
Schedule a game between B and D
Misha 2021-05-19 22:02:05
We play B vs D. Either $(A,B,D)$ or $(B,D,E)$ will be the subsequence we want.
Misha 2021-05-19 22:02:31
(The color varies depending on which case we're in, but the two cases are otherwise identical.)
Misha 2021-05-19 22:02:36
What about cases (i) and (iv)? Here, we'll need two more games.
CircleInvert 2021-05-19 22:03:41
B vs C and C vs D
artistic_girl 2021-05-19 22:03:41
compare C with B and D
Misha 2021-05-19 22:03:49
We play B vs C and C vs D. In case (i), we either have $A \prec B \prec C$, or $B \succ C \succ D$, or $C \prec D \prec E$. In case (iv), we get the opposite of one of these.
Misha 2021-05-19 22:04:17
Maybe an easy way to see this case is: we've ended up playing 4 games - A vs B, B vs C, C vs D, and D vs E.
Misha 2021-05-19 22:04:32
If we don't get the subsequence we wanted, the arrows must alternate red-blue-red-blue
Misha 2021-05-19 22:04:51
but we're looking at cases (i) and (iv) - where the first and last arrows are the same color!
Misha 2021-05-19 22:05:10
There's more to the problem: verifying that 3 games are not enough. For reasons of time, I'll skip this; there's clean and less clean ways to do the casework. I'll just say that a common pitfall was to assume that the first two games have a player in common - the strategy above shows that other approaches are valid!
Misha 2021-05-19 22:05:46
That's part (a) and I want to pause here for a bit for questions, just to make sure everything makes sense before we make everything more complicated. Everyone with me?
Misha 2021-05-19 22:06:46
Problem 6(b). Now, suppose you want to find either a subsequence $(A_1, A_2, A_3)$ such that $A_1 \prec A_2 \prec A_3$, or a subsequence $(A_1, A_2, \dots, A_n)$ such that $A_1 \succ A_2 \succ \dots \succ A_n$.
Misha 2021-05-19 22:06:51
Show that you can always do this in less than $10n$ games. (As a challenge, how much can you improve this bound?)
Misha 2021-05-19 22:07:01
Problem 6(b) solution.
Misha 2021-05-19 22:07:07
We gave you $10n$ games for flexibility, but there are shorter solutions. I'll present the best bound anyone actually found (though it's possible to do better).
Misha 2021-05-19 22:07:47
One way to try to find the long subsequence we want is to keep trying to extend a sequence. Have a camper play against other campers to their left until they win a game, then repeat this with the loser of that game. After a while, we get a picture like the following:
Misha 2021-05-19 22:07:53
//cdn.artofproblemsolving.com/images/7/0/0/70051b61985abdb906a10fbe873e4690c050c628.png
Misha 2021-05-19 22:08:27
(So for example 1 lost to 2, so 1 kept playing and lost to 3, so 1 kept playing and finally beat 4.)
artistic_girl 2021-05-19 22:09:28
this can go on forever
Misha 2021-05-19 22:09:38
Okay so we have a problem if lots of people keep losing a lot.
Misha 2021-05-19 22:09:40
So we need a backup plan to get the long subsequence we want a different way. Can anyone spot some games we can play in the diagram above whose outcomes are more or less forced?
BobsonJoe 2021-05-19 22:11:19
2, 3 and 3, 4
Batee5a 2021-05-19 22:11:19
3,4,6?
Misha 2021-05-19 22:11:29
Campers with a blue arrow out of them must beat everyone to their right, or else we get the three-camper subsequence we wanted. For example, camper 3 must beat camper 5, or else we get $1 \prec 3 \prec 5$.
Misha 2021-05-19 22:11:58
That's the "almost forced" games I mean: those outcomes are fixed, or else we're done immediately.
Misha 2021-05-19 22:12:04
With this in mind, what's a long "backup" sequence of red arrows we can force?
Misha 2021-05-19 22:12:33
(Still in the same diagram if you like, or you can try for a general claim.)
Misha 2021-05-19 22:14:44
Well, this is tricky, especially late at night - so let me spoil the secret for you. Here's the diagram again:
Misha 2021-05-19 22:14:45
https://www.mathcamp.org/files/yearly/2021/mathjams/p6-diagram-3.png
Misha 2021-05-19 22:14:56
Take all the "skipped" campers with a blue arrow pointing out of them, and have them play in order! If any of the outcomes give us another blue arrow, we're done. Otherwise, we get a long subsequence with $A_1 \succ A_2 \succ A_3 \succ \dots$: in this case, $2 \succ 3 \succ 5 \succ 8 \succ 9$.
Misha 2021-05-19 22:15:14
so that's saying "play 2 vs 3, 3 vs 5, 5 vs 8, 8 vs 9" in this case.
artistic_girl 2021-05-19 22:16:22
they all have to be red
Misha 2021-05-19 22:16:34
Exactly! If one of those games gives us a blue arrow, we're done immediately.
Misha 2021-05-19 22:17:21
So we have two ways to win with a long subsequence. How many turns will this take?
Misha 2021-05-19 22:17:41
(Either we get a long subsequence right away, or we skip lots of campers and get a long "backup" subsequence.)
Misha 2021-05-19 22:19:51
So I'm seeing a good partial answer here that's missing one step. Think of it this way: we're happy if we get $n-1$ red arrows, or we can execute our backup plan if we get $n-1$ blue arrows. How many games until one of these happens?
artistic_girl 2021-05-19 22:20:56
2n
peace09 2021-05-19 22:20:56
$2n-2?$
Misha 2021-05-19 22:21:10
Around $2n$, if we don't worry about the constants. (Actually, $2n-3$ is enough.)
Misha 2021-05-19 22:21:48
But that doesn't mean that $2n-3$ games are all we need! What else is missing?
BobsonJoe 2021-05-19 22:22:32
$n$ games for the backup plan
Gogobao 2021-05-19 22:22:32
you still need n more games
Misha 2021-05-19 22:23:01
Well, $n-1$ more games - there are $n-1$ $\succ$ signs between $n$ campers.
Misha 2021-05-19 22:23:09
But who cares about the constant, anyway.
Misha 2021-05-19 22:23:20
So this strategy's worst case is $3n-4$ games total!
conantwiz2023 2021-05-19 22:24:15
whats the runtime for the fastest known strategy?
Misha 2021-05-19 22:24:25
(The best bound I know of is around $\frac83 n$, which involves doing this strategy at the same time as a mirror image of it, and playing them off against each other to take a shortcut. The only lower bound I know is $2n$, so we're not sure what the best possible answer is!)
Misha 2021-05-19 22:24:47
Alright, now on to the last part of the last problem!
Misha 2021-05-19 22:24:54
Problem 6(c). Next, the campers come together in an unorganized crowd, and decide to hold a Two-Player Castlefall (TPC) competition. TPC is a highly strategic and complex game, so it cannot be reduced to a single skill level: in TPC, it is possible that $A \prec B$ and $B \prec C$, but $A \succ C$.
Misha 2021-05-19 22:25:00
You want to find $n$ campers $A_1, A_2, \dots, A_n$ such that whenever $i < j$, $A_i \prec A_j$. How can you minimize the number of TPC games required in the worst case?
Misha 2021-05-19 22:25:07
Problem 6(c) solution.
Misha 2021-05-19 22:25:13
I want to point that two things change in this problem. First, skill in TPC is not transitive, so we have to play $\binom n2$ games just to verify that $n$ campers are sorted by skill. Second, no more "subsequences": the campers are in an "unorganized crowd", and we can reorder them however we like!
Misha 2021-05-19 22:25:25
So forget most things that happened in part (a) and (b)
Misha 2021-05-19 22:25:41
Here's a kind of setup that will work for us. Take a camper $X$ and have them play LOTS AND LOTS of games. Either $X$ wins many of them, or $X$ loses many of them; these are symmetric, so let's say $X$ wins lots of games. How do we want to use this?
SHZhang 2021-05-19 22:27:05
Use the campers losing against X to build a sequence, then add X to it
Misha 2021-05-19 22:27:14
We want to find $n-1$ campers $A_1, A_2, \dots, A_{n-1}$ among the campers $X$ beats. Then $X$ can be camper $A_n$, because $X$ beats each of $A_1$ through $A_{n-1}$.
Misha 2021-05-19 22:27:42
(If $X$ lost a bunch of games, we'd find $n-1$ campers $A_2, \dots, A_n$ among the people that beat $X$, and then $X$ could be camper $A_1$. Symmetric!)
Misha 2021-05-19 22:27:53
To properly analyze this strategy, we have to keep track of two things. Let's say that finding a group of $k$ campers ordered by skill took us $N_k$ games involving $C_k$ campers; we want to find a formula for $N_k$, but we'll need to know $C_k$ as well.
Misha 2021-05-19 22:28:15
To find $C_k$ campers that either all lose to $X$ or all beat $X$, we should play $2C_k-1$ more games (involving $C_{k+1} = 2C_k$ campers total). The total number of games is $N_{k+1} = N_k + 2C_k - 1$. This gives us a recurrence to solve for $N_k$ and $C_k$. First of all, what's $C_k$?
yofro 2021-05-19 22:30:10
$2^{k-1}$
Misha 2021-05-19 22:30:17
We start with $C_1 = 1$ camper necessary to find a group of $k=1$ campers. Then $C_{k+1} = 2C_k$, so it doubles each time: we have $C_k = 2^{k-1}$.
Misha 2021-05-19 22:30:32
(I saw several $2^k$ answers, but watch out for the base case.)
peace09 2021-05-19 22:30:51
Where did we find that $C_{k+1}=2C_k?$
Misha 2021-05-19 22:31:08
That's our strategy for playing lots of games involving $X$ at work!
Misha 2021-05-19 22:31:22
We need to wait until $X$ either loses to $C_k$ campers or beats $C_k$ campers.
Misha 2021-05-19 22:31:48
So we should have $X$ play against $2C_k - 1$ other campers, and then the total number of campers together with $X$ is $2C_k$.
Misha 2021-05-19 22:32:04
Anyway, after all that work, our other recurrence is now $N_{k+1} = N_k + 2^k - 1$.
Misha 2021-05-19 22:32:23
That's going to tell us the number of games to find a group of $k$ campers ordered by skill.
Misha 2021-05-19 22:32:34
This one's a bit trickier, and we already got a lot of practice solving recurrence relations way back in Question 2. So I'm just going to tell you that $N_k = 2^k - k - 1$; we can prove this by induction. To find a group of $n$ campers ordered by skill, we'll need $2^n-n-1$ games.
Misha 2021-05-19 22:33:06
If you're familiar with Ramsey's theorem, you may recognize this argument! A result called "Ramsey's theorem for tournaments" tells us that if you play all possible games between $2^{n-1}$ campers, we find the group we wanted. But just citing this theorem gives a worse bound: there are $\binom{2^{n-1}}{2}$ possible games between those campers, which is closer to $4^n$ than $2^n$.
Misha 2021-05-19 22:33:32
Anyway, that wraps up 6(c), which I went through a bit quicker because we're 90 minutes late.
Misha 2021-05-19 22:33:43
Are there any other questions about problem 6?
conantwiz2023 2021-05-19 22:33:58
is $2^n - n - 1$ the best known algorithm for the problem?
Misha 2021-05-19 22:34:16
It's the best thing I knew writing the problem, and the best thing anyone came up with!
yofro 2021-05-19 22:34:24
does this problem have a name?
Misha 2021-05-19 22:34:36
Find the optimal answer and prove it's optimal, and we can name it after you!
Misha 2021-05-19 22:35:15
There is an exponential lower bound, too, which comes from Ramsey's theorem.
Misha 2021-05-19 22:35:47
Looks like that's it for questions about problem 6, so it's back over to MiraBernstein to wrap things up!
MiraBernstein 2021-05-19 22:36:02
OK, guys, it's very late!
MiraBernstein 2021-05-19 22:36:17
I promised I'd do 5c if people still wanted, and I'm still game.
MiraBernstein 2021-05-19 22:36:27
But I can also just post the solution in the same place as this Math Jam
MiraBernstein 2021-05-19 22:36:39
If I get 5 yes'es, I'll do it now.
MiraBernstein 2021-05-19 22:36:57
Oh wow!
gordonhero 2021-05-19 22:37:12
yes
peace09 2021-05-19 22:37:12
YES
Supermath123 2021-05-19 22:37:12
yes!
Jing.Han 2021-05-19 22:37:12
yes
JamesSohigian 2021-05-19 22:37:12
Yes
w_aops 2021-05-19 22:37:12
yes
Batee5a 2021-05-19 22:37:12
Yes!
gordonhero 2021-05-19 22:37:12
yes'es
GeronimoStilton 2021-05-19 22:37:12
sure
summersunlight 2021-05-19 22:37:12
yes
MiraBernstein 2021-05-19 22:37:28
I eman 5 yes'es from distinct people
MiraBernstein 2021-05-19 22:37:37
OK, let's do this thing!!
MiraBernstein 2021-05-19 22:37:56
We want to find the expected number of tests for our strategy.
MiraBernstein 2021-05-19 22:38:03
But you surely forgot what our strategy is
MiraBernstein 2021-05-19 22:38:08
So let's review quickly
MiraBernstein 2021-05-19 22:38:23
Problem 5c (paraphrased). A city wants to test 10,000 people. You know that each person has a 5% chance of being sick, but you don’t know the exact number of sick people. This time, you don’t have to design the tests in advance (as in Part b); you can base each test on your previous results (as in Part a).

Find a strategy to reduce the number of tests that need to be done; it doesn't need to be provably optimal, but do your best. On average, how many tests will be required?
MiraBernstein 2021-05-19 22:38:44
Best strategy that we know:
MiraBernstein 2021-05-19 22:38:55
First, divide your 10000 people into 625 groups of 16. We’ll call these 625 groups the Big Groups..

(BTW, by “groups” I just mean “sets”; this has nothing to do with group theory. )

For each Big Group, do the following:

$\quad$* Let $G_0$ be the entire group.

$\quad$* Do one test on all of $G_0$:

$\qquad$* If it’s negative, you’re done with this Big Group. Go on to the next one.

$\qquad$* If it’s positive, use the approach from Lemma 1 in 5a (binary search) to find just one sick person in $G_0$.
MiraBernstein 2021-05-19 22:39:13
There may be other sick people in our Big Group $G_0$. So let’s remove the sick person we just, and call the remaining group of 15 people $G_1$.
MiraBernstein 2021-05-19 22:39:21
Now we go back and repeat all the preceding steps with $G_1$:

$\quad$* Do one test on all of $G_1$:
$\qquad$* If it’s negative, you’re done with this Big Group. Go on to the next one.
$\qquad$* If it’s positive, find one sick person in $G_1$.
MiraBernstein 2021-05-19 22:39:43
OK, say we’ve found a sick person in $G_1$. We form $G_2$ by removing the sick person we just found from $G_1$. And then we just keep going in the same way:

$\quad$* Do a pooled test on all of $G_i$, for whichever $i$ you’re on.
$\qquad$* If it’s negative, you’re done with this Big Group. Go on to the next one.
$\qquad$* If it’s positive:
$\quad\qquad$* Find one sick person in $G_i$.
$\quad\qquad$* Form $G_{i+1}$ by removing this sick person from $G_i$.
$\quad\qquad$* Do it all again for $G_{i+1}$.
MiraBernstein 2021-05-19 22:39:59
Once some $G_i$ tests negative, we can be sure that we’ve found and removed all the sick people from our original big group $G_0$. So we move on to the next Big Group.



We go through all the Big Groups this way and we’re done. That’s the strategy.
MiraBernstein 2021-05-19 22:40:24
Any questions about what the strategy is, before we analyze it?
Batee5a 2021-05-19 22:40:55
Do we have to keep the inefficiency from the repeated tests so we can guarantee the average required tests?
MiraBernstein 2021-05-19 22:41:18
That's exactly right. This strategy is very easy to make more efficient!
MiraBernstein 2021-05-19 22:41:32
For instance, why are we retesting all 15 people in $G_1$ as if we know nothing about them? Most likely, we’ve already ruled some of them out when we were looking for our first sick person in $G_0$.



For instance, if the first group of 8 that we tested came out negative, why include these 8 people in $G_1$?
MiraBernstein 2021-05-19 22:41:46
So it’s clear that this strategy can be improved, possibly by a lot.

The problem is, adding all that extra complexity makes it more difficult to compute the expected number of tests that will be required. This is why all three people who came up with this strategy chose to keep it as simple as possible.
MiraBernstein 2021-05-19 22:42:03
(Sorry, I'm repeating some stuff I said already, but it's good to have the context)
MiraBernstein 2021-05-19 22:42:15
So let’s compute the expected number of tests we’ll need. Actually, we’ll be computing not the exact expectation, but just an upper bound on it.
MiraBernstein 2021-05-19 22:42:25
From here one, this is new stuff.
MiraBernstein 2021-05-19 22:43:06
So in this strategy, there are two types of tests:

1) Tests that tell you that you’re done with your current Big Group and can move on to the next one.

2) Tests that you use to actually find a sick person.



How many tests of Type 1 are there?
gordonhero 2021-05-19 22:43:30
625
yofro 2021-05-19 22:43:30
625
summersunlight 2021-05-19 22:43:48
625
MiraBernstein 2021-05-19 22:43:50
In each Big Group, we keep removing sick people one by one, until we get to some $G_i$ that tests negative. So each Big Group will require one final negative test to clear it, after all the sick people have been removed.



That is the test of Type 1! (If the Big Group had no sick people to begin with, then this will be its only test.)



So there’s exactly one test of Type 1 per Big Group, which makes 625 such tests total.
MiraBernstein 2021-05-19 22:44:10
By the way, what can we say about the results of all these 625 tests?
gordonhero 2021-05-19 22:44:40
they're all negative
MiraBernstein 2021-05-19 22:44:42
Right, it’s important to notice that Type 1 tests are all negative -- they’re the ones that finally clear each Big Group. All positive tests are automatically of Type 2.
MiraBernstein 2021-05-19 22:45:01
To count the tests of Type 2, we’ll look at each sick person in turn. Remember, in this strategy we’re always looking for one sick person at a time!
MiraBernstein 2021-05-19 22:45:13
So let’s focus on just one sick person. How many tests did we do that led us to this particular person ?
yofro 2021-05-19 22:46:04
Depends on its position in the binary tree
MiraBernstein 2021-05-19 22:46:09
True!
gordonhero 2021-05-19 22:46:14
up to 4
MiraBernstein 2021-05-19 22:46:33
Almost! You need to count the original test that told you that $G_i$ still had sick people in it.
MiraBernstein 2021-05-19 22:46:42
To find the first sick person in our Big Group $G_0$, we needed 5 tests of Type 2: we tested the whole group (16 people), then 8 people, then 4, then 2, then 1.
MiraBernstein 2021-05-19 22:47:06
(Remember, the original test of the whole group $G_0$ is of Type 2. The 625 tests of Type 1 are the ones that tell us that we’re done with a Big Group, not the ones that tell us we need to look further.)
MiraBernstein 2021-05-19 22:47:20
$G_1$ has 15 people, so we saw that finding the sick person will take either 4 or 5 tests (including the original test of all of $G_1$). So on average, we’ll need a little less than 5 tests of Type 2, but definitely at most 5.
MiraBernstein 2021-05-19 22:47:34
Finding the next sick person, at the $G_2$ stage, will take even fewer tests on average -- and still definitely fewer than 5.
MiraBernstein 2021-05-19 22:47:43
So no matter at what stage we found the particular sick person we’re focusing on right now, it took us at most 5 tests of Type 2 to find them.
MiraBernstein 2021-05-19 22:47:52
Say we have $S$ sick people total (in all the Big Groups together). What is an easy upper bound on the number of Type 2 tests that we’ll need to find them all?
w_aops 2021-05-19 22:48:58
5 * S
summersunlight 2021-05-19 22:49:20
5xs
MiraBernstein 2021-05-19 22:49:31
Right: Finding each sick person requires at most 5 tests of Type 2, for a maximum of $5S$ tests of Type 2.
MiraBernstein 2021-05-19 22:49:40
Now what’s the expected number of sick people, if each person out of 10,000 has a 5% chance of being sick?
w_aops 2021-05-19 22:50:06
500
peace09 2021-05-19 22:50:06
500
yofro 2021-05-19 22:50:06
$500$
w_aops 2021-05-19 22:50:55
so 2500 Type 2 tests maximum
MiraBernstein 2021-05-19 22:51:12
Yup, on average we expect 500 sick people. Each of these will require at most 5 tests of Type 2. Plus we have the 625 final tests of Type 1, one for each Big Group. So what can we say about the expected total number of tests?
gordonhero 2021-05-19 22:51:45
3125
yofro 2021-05-19 22:51:45
3125
yofro 2021-05-19 22:51:49
$\le 3125$
MiraBernstein 2021-05-19 22:52:00
To be clear: we don’t know the expected number of tests exactly, but we know it is at most



$625 + 5 \cdot 500 = 3125$.
MiraBernstein 2021-05-19 22:52:08
(Finding the maximum of an expectation may sound a little weird. What we’re saying is: the expected number of tests under this strategy is some number $X$. We don’t know what $X$ is, but we know $X <= 3125$.)
MiraBernstein 2021-05-19 22:52:18
Believe it or not, even this upper bound is better than the expected number of tests in any other strategy that anyone came up with!
peace09 2021-05-19 22:52:37
Really?
MiraBernstein 2021-05-19 22:52:45
Weird, right? I found it very unintuitive!
MiraBernstein 2021-05-19 22:52:52
Even with all the inefficiency.
MiraBernstein 2021-05-19 22:52:55
One last thing to discuss is why we chose 16 as the size of our Big Groups. If your big Groups are of size $N$, then what’s the upper bound you get using the reasoning above?
MiraBernstein 2021-05-19 22:53:30
(think about where the 625 came from, where the 5 came from, and where the 500 came from)
peace09 2021-05-19 22:53:37
Maximal power of two!
MiraBernstein 2021-05-19 22:53:42
No, the maximum is 32
peace09 2021-05-19 22:54:27
??? 16 is the largest power of 2 that divides 10,000
MiraBernstein 2021-05-19 22:55:25
@peach09: Oh, yes, that's true. But you could do groups of size 32 and one group of size 16.
MiraBernstein 2021-05-19 22:55:42
The formula for the upper bound is $\lceil\frac{10000}{N}\rceil + 500 (\lceil\log_2 N\rceil + 1)$.
MiraBernstein 2021-05-19 22:55:54
People got close to this, but not quite
MiraBernstein 2021-05-19 22:56:23
$\lceil\frac{10000}{N}\rceil$ is the Type 1 tests
MiraBernstein 2021-05-19 22:56:37
The rest are the type 2 tests
MiraBernstein 2021-05-19 22:56:43
Here’s the graph of this function for $1\leq N \leq 32$:
https://www.mathcamp.org/files/yearly/2021/mathjams/QQ21_5c_1.png
MiraBernstein 2021-05-19 22:56:55
As you can see, the upper bound is minimized when $N=16$. (It’s possible that the actual expected value is minimized for a different $N$, but we don’t have a way to compute that at present.)
MiraBernstein 2021-05-19 22:57:12
I want to spend a tiny bit of time talking about other strategies, but first, any questions about this one?
gordonhero 2021-05-19 22:57:33
how do you even think this up?
MiraBernstein 2021-05-19 22:57:43
Good question! It was not a common answer.
MiraBernstein 2021-05-19 22:57:53
Other good strategies were much more common.
MiraBernstein 2021-05-19 22:57:59
This one just happened to beat them!
MiraBernstein 2021-05-19 22:58:03
So to recap, this “3125 strategy” is the best one we know of for now. Of course, all the improvements that you guys suggested earlier would make it even better, but we don’t know how to measure the improvement.
MiraBernstein 2021-05-19 22:58:14
In case you’re curious, the next best strategy that anyone came up with requires about 3196 tests on average. It is very complicated: you have to split your groups not in half but into different proportions each time. This was our own best strategy, and one of the applicants came up with it as well.
MiraBernstein 2021-05-19 22:58:23
I wish I could go over more of the strategies that different people came up with -- there were so many good ones! But I’ve been talking for too long already. So I leave you with the following challenge:
MiraBernstein 2021-05-19 22:58:31
If you solved this problem and came up with a strategy that seems to beat 3125, then we promise you made a mistake somewhere. (We checked all the solutions very carefully; many of the errors were very subtle.) See if you can figure out where the error was!
MiraBernstein 2021-05-19 22:58:39
If you came up with a strategy that doesn’t beat 3125, but intuitively feels like it should , figure out why it’s actually less efficient. For some of the best strategies, this is not so easy to understand: I still haven’t completely wrapped my mind around it…
MiraBernstein 2021-05-19 22:58:50
(One last thing to remember as you think about this: the optimal strategy depends on the probability of each person being sick. For 5%, this one happens to be the best strategy we know, but other strategies will certainly beat it for other probabilities.)
MiraBernstein 2021-05-19 22:59:15
Whew, that's it for #5, though there's lots more to tell!
MiraBernstein 2021-05-19 22:59:22
Thanks for sticking it out for so long!
MiraBernstein 2021-05-19 22:59:40
If we didn't get to your question, you can also post in the Mathcamp forum here on AoPS, at http://www.artofproblemsolving.com/community/c135_mathcamp - the Mathcamp staff will post replies, and you'll get student opinions, too.
MiraBernstein 2021-05-19 22:59:48
We hope you enjoyed this year's QQ, and stay tuned for the 2022 Quiz (and the summer 2022 application) coming up in January. I'll stick around for another five minutes and answer non-Quiz questions about the program and the application process in case anybody has questions about Mathcamp itself.



Thanks again, everybody - good night!
yofro 2021-05-19 23:00:08
But is there any strategy that is better asymptotically? Like something like $O(\log\log n)$ instead of $O(\log n)$
MiraBernstein 2021-05-19 23:00:13
Not that we know of.
MiraBernstein 2021-05-19 23:00:30
But again, it also depends on the probability of being sick
GeronimoStilton 2021-05-19 23:00:37
Thank you!
HollyFalcon 2021-05-19 23:00:37
Good night!
yofro 2021-05-19 23:00:37
Thanks, bye!
JamesSohigian 2021-05-19 23:00:37
Thanks
peace09 2021-05-19 23:00:55
TYSM!
Enthurelx 2021-05-19 23:00:55
Thanks
Lilavigne 2021-05-19 23:01:02
Thank you!
gordonhero 2021-05-19 23:01:13
Thank you!!
w_aops 2021-05-19 23:01:13
Thank you!
MiraBernstein 2021-05-19 23:01:16
Thank you guys!
MiraBernstein 2021-05-19 23:01:26
I admire your attention span!
AngelBell 2021-05-19 23:02:21
If you have further questions for Mathcamp, you can contact them at https://www.mathcamp.org/contact_us/

Copyright © 2024 AoPS Incorporated. This page is copyrighted material. You can view and print this page for your own use, but you cannot share the contents of this file with others.